+7 495 120-13-73 | 8 800 500-97-74

(для регионов бесплатно)

Содержание

Цепь переменного тока с индуктивным сопротивлением.

Индуктивность в цепи переменного тока

 Прохождение электрического тока по проводнику или катушке сопровождается появлением магнитного поля. Рассмотрим электрическую цепь переменного тока (рис. 57, а), в которую включена катушка индуктивности, имеющая небольшое количество витков проволоки сравнительно большого сечения, активное сопротивление которой можно считать практически равным нулю.
Под действием э. д. с. генератора в цепи протекает переменный ток, возбуждающий переменный магнитный поток. Этот поток пересекает «собственные» витки катушки и в ней возникает электродвижущая сила самоиндукции где L — индуктивность катушки;
 — скорость изменения тока в ней.
Электродвижущая сила самоиндукции, согласно правилу Ленца, всегда противодействует причине, вызывающей ее. Так как э. д. с. самоиндукции всегда противодействует изменениям переменного тока, вызываемым э. д. с. генератора, то она препятствует прохождению переменного тока.
При расчетах это учитывается по индуктивному сопротивлению, которое обозначается XL и измеряется в омах.

Таким образом, индуктивное сопротивление катушки XL, зависит от величины э. д. с. самоиндукции, а следовательно, оно, как и э. д. с. самоиндукции, зависит от скорости изменения тока в катушке (от частоты ω) и от индуктивности катушки L

где XL — индуктивное сопротивление, ом;
ω — угловая частота переменного тока, рад/сек;
L — индуктивность катушки, гн.
Так как угловая частота переменного тока ω = 2πf, то индуктивное сопротивление
XL = 2πf L,    (59)

где f — частота переменного тока, гц.

Пример. Катушка, обладающая индуктивностью L = 0,5 гн, присоединена к источнику переменного тока, частота которого f = 50 гц. Определить:
1) индуктивное сопротивление катушки при частоте f = 50 гц;
2) индуктивное сопротивление этой катушки переменному току, частота которого f = 800 гц.
Решение. Индуктивное сопротивление переменному току при f = 50 гц

XL = 2πf L = 2 · 3,14 · 50 · 0,5 = 157 

ом.

При частоте тока f = 800 гц

XL = 2πf L = 2 · 3,14 · 800 · 0,5 = 2512 ом.

Приведенный пример показывает, что индуктивное сопротивление катушки повышается с увеличением частоты переменного тока, протекающего по ней. По мере уменьшения частоты тока индуктивное сопротивление убывает. Для постоянного тока, когда ток в катушке не изменяется и магнитный поток не пересекает ее витки, э. д. с. самоиндукции не возникает, индуктивное сопротивление катушки XL равно нуло. Катушка индуктивности для постоянного тока представляет собой лишь сопротивление Выясним, как изменяется з. д. с. самоиндукции, когда по катушке индуктивности протекает переменный ток.
Известно, что при неизменной индуктивности катушки э. д. с. самоиндукции зависит от скорости изменения силы тока и она всегда направлена навстречу причине, вызвавшей ее.
На графике (рис. 57, в) переменный ток показан в виде синусоиды (сплошная линия). В первую четверть периода сила тока возрастает от нулевого до максимального значения. Электродвижущая сила самоиндукции ес, согласно правилу Ленца, препятствует увеличению тока в цепи. Поэтому на графике (пунктирной линией) показано, что ес в это время имеет отрицательное значение. Во вторую четверть периода сила тока в катушке убывает до нуля. В это время э. д. с. самоиндукции изменяет свое направление и увеличивается, препятствуя убыванию силы тока. В третью четверть периода ток изменяет свое направление и постепенно увеличивается до максимального значения; э. д. с. самоиндукции имеет положительное значение и далее, когда сила тока убывает, э. д. с. самоиндукции опять меняет свое направление и вновь препятствует уменьшению силы тока в цепи. Из сказанного следует, что ток в цепи и э. д. с. самоиндукции не совпадают по фазе. Ток опережает э. д. с. самоиндукции по фазе на четверть периода или на угол φ = 90°.
Необходимо также иметь в виду, что в цепи с индуктивностью, не содержащей г, в каждый момент времени электродвижущая сила самоиндукции направлена навстречу напряжению генератора U. В связи с этим напряжение и э. д. с. самоиндукции ес также сдвинуты по фазе друг относительно друга на 180°.
Из изложенного следует, что в цепи переменного тока, содержащей только индуктивность, ток отстает от напряжения, вырабатываемого генератором, на угол φ = 90° (на четверть периода) и опережает э. д. с. самоиндукции на 90°. Можно также сказать, что в индуктивной цепи напряжение опережает по фазе ток на 90°.
Построим векторную диаграмму тока и напряжения для цепи переменного тока с индуктивным сопротивлением. Для этого отложим вектор тока 
I
 по горизонтали в выбранном нами масштабе (рис. 57, б.)
Чтобы на векторной диаграмме показать, что напряжение опережает по фазе ток на угол φ = 90°, откладываем вектор напряжения U вверх под углом 90°. Закон Ома для цепи с индуктивностью можно выразить так: Следует подчеркнуть, что имеется существенное отличие между индуктивным и активным сопротивлением переменному току.
Когда к генератору переменного тока подключена активная нагрузка, то энергия безвозвратно потребляется активным сопротивлением.
Если же к источнику переменного тока присоединено индуктивное сопротивление 
r
 = 0, то его энергия, пока сила тока возрастает, расходуется на возбуждение магнитного поля. Изменение этого поля вызывает возникновение э. д. с. самоиндукции. При уменьшении силы тока энергия, запасенная в магнитном поле, вследствие возникающей при этом э. д. с. самоиндукции возвращается обратно генератору.
В первую четверть периода сила тока в цепи с индуктивностью возрастает и энергия источника тока накапливается в магнитном поле. В это время э. д. с. самоиндукции направлена против напряжения.
Когда сила тока достигнет максимального значения и начинает во второй четверти периода убывать, то э. д. с. самоиндукции, изменив свое направление, стремится поддержать ток в цепи. Под действием э. д. с. самоиндукции энергия магнитного поля возвращается к источнику энергии — генератору.
Генератор в это время работает в режиме двигателя, преобразуя электрическую энергию в механическую.
В третью четверть периода сила тока в цепи под действием э. д. с. генератора увеличивается, и при этом ток протекает в противоположном направлении. В это время энергия генератора вновь накапливается в магнитном поле индуктивности.
В четвертую четверть периода сила тока в цепи убывает, а накопленная в магнитном поле энергия при воздействии э. д. с. самоиндукции вновь возвращается генератору.
Таким образом, в первую и третью четверть каждого периода генератор переменного тока расходует свою энергию в цепи с индуктивностью на создание магнитного поля, а во вторую и четвертую четверть каждого периода энергия, запасенная в магнитном поле катушки в результате возникающей э. д. с. самоиндукции, возвращается обратно генератору.
Из этого следует, что индуктивная нагрузка в отличие от активной в среднем не потребляет энергию, которую вырабатывает генератор, а в цепи с индуктивностью происходит «перекачивание» энергии от генератора в индуктивную нагрузку и обратно, т.
е. возникают колебания энергии.
Из сказанного следует, что индуктивное сопротивление является реактивным. В цепи, содержащей реактивное сопротивление, происходят колебания энергии от генератора к нагрузке и обратно.

Reactance

http://en.wikipedia.org  Wikipedia, свободная энциклопедия

Реактивное сопротивление — это мнимая часть импеданса (импедансом называется полное (комплексное) сопротивление цепи переменного тока), которая показывает меру противодействия синусоидальному переменному току. Реактивное сопротивление возникает в присутствии индуктивности и ёмкости в цепи, и обозначается символом X; единица СИ — Ом.

 

(В этом разделе знак тильда (~) будет использован для обозначения векторов или комплексных величин, а буквы без дополнительных знаков обозначают модули векторов соответствующих величин, а также скалярные величины.)

Для определения импеданса   требуется как реактивное сопротивление X, так и резистивное (активное) сопротивление R. Несмотря на то, что в некоторых обстоятельствах реактивное сопротивление может доминировать, требуется хотя-бы приблизительное знание активного сопротивления  для определения импеданса. 

Как модуль, так и фаза  импеданса зависят от обоих сопротивлений – и от активного и от реактивного:

Модуль импеданса — это отношение амплитуд напряжения и тока, тогда как фаза - это разница между фазами напряжения и тока. 

  • Если X>0 говорят, что реактивное сопротивление является индуктивным
  • Если X=0 говорят, что импеданс чисто резистивный (активный)
  • Если X<0 говорят, что реактивное сопротивление является ёмкостным

Физическое значение

Определение соотношений между током и напряжением требует знания, как активного, так и реактивного сопротивлений. Реактивное сопротивление само по себе даёт только ограниченную физическую информацию об электрическом устройстве или электрической цепи:

  1. Величина реактивного сопротивления показывает меру противодействия цепи только переменному току, и зависит от частоты переменного тока.  
  2. Положительное реактивное сопротивление подразумевает, что фаза напряжения опережает фазу тока, в то время как отрицательное реактивное сопротивление подразумевает, что фаза напряжения отстаёт от фазы тока.
  3. Нулевое реактивное сопротивление подразумевает, что ток и напряжение совпадают по фазе и наоборот, если реактивное сопротивление не равно нулю, тогда существует разность фаз между напряжением и током.         

Есть случаи, когда  в цепи есть реактивные элементы, но результируюшее реактивное сопротивление цепи равно нулю, для примера: резонанс в RLC-цепи случается, когда реактивные импедансы ZC и ZL взаимоуничтожаются. Это значит, что импеданс имеет фазу, равную нулю (специфический пример нулевого реактивного сопротивления для случая 3. выше).

Ёмкостное реактивное сопротивление

Ёмкостное реактивное сопротивление Xc обратнопропорционально частоте сигнала  и ёмкости C.

Ёмкостной элемент называется конденсатором. Конденсатор состоит из двух проводников, отделённых друг от друга изолятором, тоесть диэлектриком.

При низких частотах или в цепи постоянного тока конденсатор разрывает (размыкает) цепь, так как ток не может течь через диэлектрик. Если к изначально разряженному конденсатору прикладывают постоянное напряжение – в начальный момент на обкладках конденсатора индуцируются заряды, электрическое поле котрых противоположно полю внешнего источника напряжения. Поэтому ток в этот начальный момент в цепи максимален. Затем потенциалы источника питания и конденсатора точно уравниваются, и ток в цепи прекращается.

Конденсатор, включённый в цепь переменного тока, будет успевать накапливать только ограниченный заряд перед тем, как разность потенциалов изменит знак на противоположный. Тоесть ток не будет успевать упасть до нуля, как в случае цепи постоянного тока. Чем выше частота, тем меньший заряд будет аккумулироваться в конденсаторе, и тем меньше конденсатор будет противодействовать внешнему току (сопротивление уменьшается).

Индуктивное реактивное сопротивление

Индуктивное реактивное сопротивление XLпрямопропорционально частоте сигнала и индуктивности L.

Индуктивный элемент представляет собой катушку индуктивности, тоесть длинный проводник, например проволока, намотанный в виде катушки. Изнутри катушка может быть пустая или содержать магнетик. Закон электромагнитной индукции Фарадея устанавливает, что ЭДС электромагнитной индукции в замкнутом контуре численно равна и противоположна по знаку скорости изменения магнитного потока через поверхность, ограниченную этим контуром. Эта ЭДС часто называется противо-ЭДС.

Если индуктивность представляет собой катушку содержащую N витков.

В общем случае ЭДС является следствием изменения магнитного потока в контуре. Но это изменение магнитного потока может иметь разные причины: движение магнита, движение другой катушки с током, изменение собственного тока контура. Последний случай носит название – явление самоиндукции, которое и лежит в основе индуктивного реактивного сопротивления.

В свою очередь противо-ЭДС вызывает в контуре индукционный ток, который направлен противоположно току источника питания. Точная форма правила Ленца: индукционный ток в контуре имеет такое направление, что созданный им магнитный поток, через контур, препятствует изменению магнитного потока, вызвавшего этот ток.

Если к изначально неподключенной катушке индуктивности подключают источник постоянного тока – в начальный момент в катушке начинает течь ток от внешнего источника. Он вызывает изменение магнитного потока. Изменение магнитного потока порождает противо-ЭДС. Противо-ЭДС вызывает противоток. Этот противоток в начальный момент равен току источника.

При низких частотах или в цепи постоянного тока катушка индуктивности проводит электрический ток беспрепятственно, и может рассматриваться как короткозамкнутый участок цепи, тоесть проводник с низким сопротивлением. Если к изначально неподключенной катушке индуктивности подключают источник постоянного тока – в начальный момент в катушке возникает противоток, равный току внешнего источника. Поэтому для идуктивного элемента в этот начальный момент результирующий ток равен нулю, а напряжение максимально. Затем токи источника и индуктивного элемента уравниваются и напряжение на индуктивном элементе становится равным нулю.

Ток в катушке индуктивности, включённой в цепь переменного тока, будет успевать возрасти только до определённого значения перед тем, как ток источника питания изменит знак на противоположный. Тоесть напряжение (на выводах катушки индуктивности) не будет успевать упасть до нуля, как в случае цепи постоянного тока. Чем выше частота, тем выше напряжение на выводах катушки индуктивности (сопротивление увеличивается).

Фазные соотношения

Фаза напряжения приложенного к чисто реактивному устройству (устройству с нулевым активным сопротивлением) отстаёт от фазы тока на Pi/2 для ёмкости и опережает фазу тока на Pi/2 для индуктивности. Необходимо отметить, что для определения соотношений между током и напряжением необходимо знать как активное, так и реактивное сопротивление.

Причина различных знаков ёмкостного и индуктивного сопротивлений заключается в определении фазной переменной импеданса.

Для реактивного элемента цепи синусоидальное напряжение на элементе сдвинуто по фазе на 90 градусов (Pi/2 радиан) относительно тока. Элемент поочерёдно то поглащает энергию из сети, то затем возвращает энергию обратно в сеть, поэтому чисто реактивное сопротивление не поглащает энергию.

 

Индуктивное сопротивлени — Энциклопедия по машиностроению XXL

Для современных источников питания дуги переменного тока падающую внешнюю характеристику получают путем искусственного увеличения индуктивного сопротивления.  [c.131]

Сварочные трансформаторы, как правило, имеют падающую внешнюю характеристику, их используют для дуговой ручной сварки и автоматической сварки под флюсом. Широко применяют трансформаторы с увеличенным магнитным рассеянием и подвижной вторичной обмоткой (типов тс и ТД). В этих трансформаторах (рис. 5.5, о) первичная I и вторичная 2 обмотки раздвинуты относительно друг друга, что обусловливает их повышенное индуктивное сопротивление вследствие появления магнитных потоков рассеяния.  [c.188]


Для плавного регулирования сварочного тока изменяют расстояние между обмотками трансформатора. При сближении обмоток (рис. 5.5, б) происходит частичное взаимное уничтожение противоположно направленных потоков рассеяния и что уменьшает индуктивное сопротивление вторичной обмотки и увеличивает сварочный ток. Минимальный сварочный ток соответствует наибольшему расстоянию между обмотками и максимальным потоком рассеяния (рис. 5.5, в).  [c.189]

Сварочные выпрямители состоят из трехфазного понижающего трансформатора /, выпрямительного блока 2, собранного из кремниевых полупроводниковых вентилей по трехфазной мостовой схеме (рис. 5.6). Падающая внешняя характеристика выпрямителя обеспечивается повышенным индуктивным сопротивлением понижающего трансформатора, у которого первичная и вторичная обмотки раздвинуты и размещены на разных концах магнитопровода (тип ВД). Плавное регулирование тока достигается перемещением подвижной первичной обмотки.  [c.189]

Например, в быстрых расчетных моделях СГ средней и малой мощности можно выделить следующие типовые блоки 1) расчет геометрии активных частей 2) расчет обмоточных данных 3) расчет ненасыщенных параметров (активных и индуктивных сопротивлений) 4) расчет магнитной цепи в установившемся режиме 5) расчет насыщенных параметров 6) расчет потерь и КПД  [c.124]

Произведение циклической частоты О) на индуктивность L называют индуктивным сопротивлением. Обозначив индуктивное сопротивление Х , запишем  [c.242]

Как и индуктивное сопротивление катушки, емкостное сопротивление конденсатора не является постоянной величиной. Его значение обратно пропорционально частоте пере-  [c.244]

Емкостное сопротивление конденсатора и индуктивное сопротивление катушки зависят от частоты (I) приложенного напряжения. Поэтому при постоянной амплитуде U колебаний напряжения амплитуда 1 , колебаний силы тока в цепи зависит от частоты (1) переменного напряжения.  [c.244]

При дальнейшем увеличении частоты индуктивное сопротивление катушки начинает превышать емкостное сопротивление  [c.245]

Если активное сопротивление провода первичной катушки мало по сравнению с его индуктивным сопротивлением, то приложенное напряжение Ui в любой момент времени примерно равно ЭДС самоиндукции, взятой с противоположным знаком.  [c.246]

Изотермический процесс 81 Изотопы 315 Изотропность 88 Изохора 82 Изохорный процесс 81 Импульс силы 40 Импульс тела 40 Индуктивное сопротивления 242 Индуктивность 190 Индуктор 196  [c.361]

Наконец, подставляя это в Fx, получим окончательно для индуктивного сопротивления следующую формулу  [c. 264]

Если увеличить все размеры по оси z в некоторое число раз (при неизменных Г), то интеграл (47,4) не изменится ). Это показывает, что полное индуктивное сопротивление крыла не изменяется по порядку величины при увеличении его размаха. Другими словами, индуктивное сопротивление, отнесенное к единице длины крыла, падает с увеличением этой длины ). В противоположность сопротивлению полная подъемная сила  [c.264]

В пределе, при стремлении размаха к бесконечности, отнесенное к единице длины индуктивное сопротивление обращается в н ль. В действительности при этом остается небольшое сопротивление, определяющееся расходом  [c.264]


Аналогичные соотношения можно получить и для силы сопротивления. Наряду с формулой Жуковского для подъемной силы полностью переносится в теорию сжимаемой жидкости также и формула (47,4) для индуктивного сопротивления крыла. Произведя в ней те же преобразования (124,3) и (124,8), получим  [c.650]

Для синхронного двигателя (СД) с электромагнитным возбуждением и пусковыми (демпферными) короткозамкнутыми обмотками уравнения электрического равновесия непосредственно вытекают из (5.7) и при переходе к индуктивным сопротивлениям, приведенным к номинальной частоте питания соо> имеют вид  [c.107]

Для практического анализа параметры СЗ целесообразно выражать в относительных единицах, отражая при этом их связь лишь с геометрией (преимущественно относительной) и свойствами материалов ЭД. В качестве базового выбрано индуктивное сопротивление гладкого 114  [c.114]

В связи со скосом потока вектор подъемной силы крыла поворачивается на тот же угол Аа, так как его направление всегда перпендикулярно к истинному направлению потока (рис. 10.76). Проекция подъемной силы крыла конечного размаха ) на направление невозмущенного потока представляет собой силу так называемого индуктивного сопротивления-.[c.100]

Таким образом, влияние конечного размаха крыла сказывается в появлении особого рода (индуктивного) сопротивления даже в случае обтекания крыла идеальной жидкостью.  [c.100]

Режим сварки при использовании трансформаторов с дросселем и трансформаторов с подмагпичивацием постоянным током регулируют путем изменения индуктивного сопротивления реактивной обдготки или участка сердечника, насыщаемого от дополнительной обмотки, питаемой no TOHJJUbur токо. г.  [c.132]

При работе трансформатора основной магнитный поток Фо, создаваемый первичной и вторичной обмотками, замыкается через магннтопровод 3. Часть магнитного потока ответвляется и замыкается вокруг обмоток через воздушное пространство, образуя потоки рассеяния и s2- Потоки рассеяния индуктируют в обмотках электродвижущую силу, противоположную основному напряжению. С увеличением сварочного тока увеличиваются потоки рассеяния и, следовательно, возрастает индуктивное сопротивление вторичной обмотки, что и создает внешнюю падающую характеристику трансформатора.[c.189]

Таким образом, в общем случае обобщенные модели могут быть представлены системами взаимодвижущихся катушек (конденсаторов). В качестве обобщенной модели будем рассматривать систему с взаимодвижущимися катушками, так как практическое применение нашли индуктивные ЭМП. Каждую катушку можно представить двухполюсником с последовательным соединением активного и индуктивного сопротивлений. К зажимам катушки можно подводить или отводить электрическую энергию. Катушки могут иметь произвольные электрические соединения друг с другом.  [c.56]

Уравнения (4.3) являются обычными дифференциальными уравнениями с вещественными постоянными коэффициентами, а в случае (o= onst они становятся линейными. Решение подобных уравнений излагается в математических справочниках и не вызывает затруднений. Однако постоянство индуктивных сопротивлений в (4.3), достигнутое при пренебрежении насыщением, приводит к большим погрешностям в решении уравнений. Учет насыщения в осях d, q осуществляется проще, чем для исходной модели ЭМП (рис. 4.1, а). Обычно насыщение учитывается раздельно по каждой из осей d. q. Для этого вводятся новые переменные в виде собственных и взаимных потокосцеплений катушек, которые связываются с токами с помощью заданных функций насыщения.  [c.86]

Таким образом, ограничение по перегрузочной способности можно задавать в различных формах, что расширяет возможности.проектировщика. Насыщенные значения Xd и х также стабильны, но имеют несколько больший диапазон изменения, так как зависят не только от геометрических размеров зазора, но и степени насыщения магннтопровода, т. е. от всех остальных геометрических размеров и режимов работы. Уменьшение кривых, индуктивных сопротивлений рассеивания и активных сопротивлений естественно и объясняется соответствующим увеличением воздушного зазора и уменьшением числа витков.  [c.207]

Зависимость индуктивного сопротивления от частоты. Хотя вы-ран ения (43.3) и (70.4) совпадают по форме, между ними имеются принципиальные отличия по су-щe твJ Электрическое сопротивление проводника при данной температуре является постоянной величиной, характеризующей проводник. Индуктивное сопротивление не является постоянной величиной, его значение прямо пропорционально частоте переменного тока. Поэтому амплитуда 1,п колебаний силы тока в проводнике индуктивностью L при постоянном значении амплитуды Um колебаний напряжения убывает обратно пропорционально частоте  [c.242]

При выполнении условия (V1.1), т. е. при равенстве индуктивного сопротивления катушки емкостному сопротивлению конденсатора, и одинаковой силе тока одинаковыми оказываются и амплитуды колебаний напряжения на конденсаторе и катушке. Колебания напряжения на катушке и конденсаторе противоположны по фазе, поэтому сумма напряжений на них при выполнении условия (71.1) в любой момент времени равна нулю. В ре-Г1ультате напряжение на активном сопротивлении при резонансе оказывается равным полному напряжению  [c.244]


Определить минимлльное значение индуктивного сопротивления, которое может быть достигнуто при заданных подъемной силе и размахе крыла 1=1.  [c.266]

Обратим внимание на формальную аналогию между формулой (123,5) и формулой (47,4) для индуктивного сопротивления тонкого крыла вместо функции Г (г) в (47,4) здесь стоит функция OiS ji). Вввиду этой аналогии для вычисления интеграла  [c.646]

В унифицированной СЗ по рис. 5.2, пригодной для ЭД разного типа, ротор представляется эквивалентными активными 21, К22 и индуктивными Х21, Х22 элементами, образующими две параллельные цепи. Для синхронного режима СД сопротивления одной из ветвей определяются наличием возбуждения, а другой — лишь его явнополюсно-стью. При отсутствии возбуждения (АД, СРД) для неявнополюсного СД, а также для гистерезисных ЭД в СЗ присутствует лишь одна ветвь ротора с сопротивлениями Кг тл Х — Последнее в зависимости от степе-Ди возбуждения и нагрузки СД может быть положительным или. отрицательным (выступая как емкостное). Намагничиваюший контур представлен в СЗ действительным индуктивным сопротивлением цепи намагничивания Хд (н) (хотя ток в нем при наличии возбуждения и не равен фактическому току XX), а введение в него в соответствии с понятием комплексной магнитной проницаемости активного сопротивления Го (т>) позволяет достаточно точно учесть также и потери в стали статора, что при обычном анализе синхронных ЭД вызывает определенные затруднения.  [c.114]

Переходя к безразмерным величинам и учитывая малость угла скоса (sinAa Aa), получаем формулу для определения так называемого коэффициента индуктивного сопротивления крыла конечного раз.маха  [c.100]


Реактивное емкостное сопротивление — Большая Энциклопедия Нефти и Газа, статья, страница 1

Реактивное емкостное сопротивление

Cтраница 1


Реактивное емкостное сопротивление и положительное активное сопротивление элементов этой цепочки могут компенсировать связь через пространственный заряд в узком диапазоне частот. Для этого контур сигнальной сетки настраивают на частоту гетеродина и параметры компенсирующей цепи R и С подбирают так, чтобы на сигнальной сетке не появлялось заметное напряжение гетеродина.  [2]

Как и активное, реактивное емкостное сопротивление оценивают в омах. Значение его зависит от емкости и частоты: чем больше емкость и частота, тем меньше сопротивление. Катушка индуктивности, включенная в цепь переменного тока, тоже обладает реактивным сопротивлением XL: ( uL2n L. Катушка с конденсатором образует колебательный контур.  [3]

Сопротивление Хс называется реактивным емкостным сопротивлением. Оно отличается от активного сопротивления R, в котором вся мощность, отдаваемая источником тока, превращается в тепло и расходуется на нагревание сопротивления. В случае же емкостного реактивного сопротивления Хс вся работа, совершаемая источником тока при заряде конденсатора, преобразуется в энергию электрического поля, а при разряде вся энергия без остатка возвращается в источник. Потому сопротивление и называют реактивным.  [4]

В диэлектрическом усилителе преобразование энергии источника питания в энергию усиливаемого сигнала производится нелинейным реактивным емкостным сопротивлением, роль которого играет конденсатор с сегнетодиэлектриком.  [5]

При необходимости длину сварочной цепи можно нарастить, однако стабильность процесса в этом случае ухудшается ввиду увеличения активного, реактивного и емкостного сопротивления сварочной цепи и снижения возможностей источника питания по отработке возмущений, связанных с изменением вылета электрода и длины дуги. Сварочные кабели запрещено при работе укладывать в бухты или наматывать на вьюшки вследствие значительного увеличения индуктивного сопротивления сварочной цепи.  [6]

Это происходит в том случае, когда в одних ветвях преобладает реактивное индуктивное сопротивление, а в других — реактивное емкостное сопротивление.  [7]

При последовательной схеме замещения измеряемое сопротивление РК ( активная составляющая Ra) равно сумме первого и второго членов правой части выражения (3.24), третий член определяет реактивное емкостное сопротивление.  [9]

В электрических цепях переменного тока при параллельном соединении реактивных сопротивлений может возникать резонанс токов. Это происходит в том случае, когда в одних ветвях преобладает реактивное индуктивное сопротивление, а в других — реактивное емкостное сопротивление.  [10]

Практически о величине емкости конденсатора ( действующей емкости) мы судим по величине тока, потребляемого конденсатором. Наличие индуктивностиЬв конденсаторе дает увеличение его действующей емкости с ростом частоты ( рис. 30), так как ток будет возрастать за счет компенсации реактивного емкостного сопротивления индуктивным сопротивлением.  [12]

В простейшем случае резонанс напряжений может быть получен в электрической цепи переменного тока при последовательном включении катушки индуктивности и конденсаторов. При этом, изменяя емкость конденсаторов при постоянных параметрах катушки, получают резонанс напряжений при неизменных значениях напряжения и индуктивности, частоты и активного сопротивления цепи. При изменении емкости С конденсаторов происходит изменение реактивного емкостного сопротивления. При этом полное сопротивление цепи также изменяется, следовательно, изменяются ток, коэффициент мощности, напряжения на катушке индуктивности, конденсаторах и активном сопротивлении катушки и активная, реактивная и полная мощности электрической цепи.  [13]

В простейшем случае резонанс напряжений может быть получен в электрической цепи переменного тока при последовательном включении катушки индуктивности и конденсаторов. При этом, изменяя емкость конденсаторов при постоянных параметрах катушки, получают резонанс напряжений при неизменных значениях напряжения и индуктивности, частоты и активного сопротивления цепи. При изменении емкости С конденсаторов происходит изменение реактивного емкостного сопротивления. При этом полное сопротивление цепи также изменяется, следовательно, изменяются ток, коэффициент мощности, напряжения на катушке индуктивности, конденсаторах и активном сопротивлении катушки и активная, реактивная и полная мощности электрической цепи.  [14]

Страницы:      1

Реактивное сопротивление — это… Что такое Реактивное сопротивление?

  • РЕАКТИВНОЕ СОПРОТИВЛЕНИЕ — величина, характеризующая сопротивление, оказываемое переменному току электрической емкостью и индуктивностью цепи (ее участка). Реактивное сопротивление синусоидальному току при последовательном соединении индуктивного и емкостного элементов… …   Большой Энциклопедический словарь

  • реактивное сопротивление — Параметр пассивного двухполюсника, равный квадратному корню из разности квадратов полного и активного электрических сопротивлений двухполюсника, взятому со знаком плюс, если электрический ток отстает по фазе от электрического напряжения, и со… …   Справочник технического переводчика

  • реактивное сопротивление КЗ — — [Я.Н.Лугинский, М.С.Фези Жилинская, Ю.С.Кабиров. Англо русский словарь по электротехнике и электроэнергетике, Москва, 1999 г.] Тематики электротехника, основные понятия EN short circuit reactance …   Справочник технического переводчика

  • реактивное сопротивление — 147 реактивное сопротивление Параметр пассивного двухполюсника, равный квадратному корню из разности квадратов полного и активного электрических сопротивлений двухполюсника, взятому со знаком плюс, если электрический ток отстает по фазе от… …   Словарь-справочник терминов нормативно-технической документации

  • реактивное сопротивление — величина, характеризующая сопротивление, оказываемое переменному току электрической ёмкостью и индуктивностью цепи (её участка). Реактивное сопротивление синусоидальному току при последовательном соединении индуктивного и ёмкостного элементов… …   Энциклопедический словарь

  • реактивное сопротивление — reaktyvioji varža statusas T sritis automatika atitikmenys: angl. reactance; reactive resistance vok. Blindwiderstand, m; imaginärer Widerstand, m; Reaktanz, f; reaktiver Widerstand, m rus. реактивное сопротивление, n pranc. réactance, f …   Automatikos terminų žodynas

  • реактивное сопротивление — reaktyvioji varža statusas T sritis Standartizacija ir metrologija apibrėžtis Kintamosios srovės grandinės varža, sudaryta iš induktyviosios ir talpinės varžų. atitikmenys: angl. reactance; reactive resistance vok. Blindwiderstand, m; Reaktanz, f …   Penkiakalbis aiškinamasis metrologijos terminų žodynas

  • реактивное сопротивление — reaktyvioji varža statusas T sritis Standartizacija ir metrologija apibrėžtis Kompleksinės elektrinės varžos menamoji dalis. atitikmenys: angl. reactance; reactive resistance vok. Blindwiderstand, m; Reaktanz, f rus. реактанс, m; реактивное… …   Penkiakalbis aiškinamasis metrologijos terminų žodynas

  • реактивное сопротивление — reaktyvioji varža statusas T sritis chemija apibrėžtis Sistemos kintamosios srovės varža, sudaryta iš induktyviosios ir talpinės varžų. atitikmenys: angl. reactance; reactive resistance rus. реактанс; реактивное сопротивление ryšiai: sinonimas –… …   Chemijos terminų aiškinamasis žodynas

  • реактивное сопротивление — reaktyvioji varža statusas T sritis fizika atitikmenys: angl. reactance; reactive resistance vok. Blindwiderstand, m; imaginärer Widerstand, m; Reaktanz, f; reaktiver Widerstand, m rus. реактанс, m; реактивное сопротивление, n pranc. réactance,… …   Fizikos terminų žodynas

  • Активное и реактивное сопротивление | Практическая электроника

    В этой статье мы поведем речь о таких параметрах, как активное и реактивное сопротивление.

    Активное сопротивление

    И начнем мы статью не с реактивного сопротивления, как ни странно, а с простого и всеми нами любимого радиоэлемента  – резистора, который, как говорят, обладает активным сопротивлением. Еще иногда его называют омическим.  Как нам говорит вики-словарь, “активный  – это деятельный, энергичный, проявляющий инициативу”. Активист готов всегда рвать и метать даже ночью. Он готов ПОЛНОСТЬЮ выложиться и потратить всю энергию во благо общества.

    То же самое можно сказать и про другие нагрузки, обладающие активным сопротивлением. Это могут быть различные нагревательные элементы, типа тэнов, а также лампы накаливания.

    Как смотреть силу тока в цепи через осциллограф

    Чем же резистор отличается от  катушки индуктивности  и конденсатора? Понятное дело, что выполняемыми функциями, но этим все не ограничивается. Итак, давайте рассмотрим самую простую схемку во всей электронике:

    На схеме мы видим генератор частоты и резистор.

    Давайте визуально посмотрим, что у нас творится в этой схеме. Для этого, как я уже сказал, нам понадобится генератор частоты

    А также цифровой осциллограф:

    С помощью него мы будем смотреть напряжение и  силу тока . 

    Что?

    Силу тока?

    Но ведь осциллограф предназначен для того, чтобы рассматривать форму сигнала напряжения? Как же мы будем рассматривать форму сигнала силы тока? А все оказывается просто). Для этого достаточно вспомнить правило шунта.

    Кто не помнит –  напомню. Имеем обыкновенный резистор:

    Что будет, если через него прогнать электрический ток?

    На концах резистора у нас будет падение напряжения. То есть, если замерить с помощью мультиметра напряжение на его концах, мультиметр покажет какое-то значение в Вольтах

    И теперь главный вопрос: от чего зависит падение напряжения на резисторе? В дело опять же вступает закон Ома для участка цепи: I=U/R. Отсюда U=IR. Мы видим зависимость от номинала самого резистора и от силы тока, текущей в данный момент в цепи. Слышите? От СИЛЫ ТОКА! Так почему бы нам не воспользоваться таким замечательным свойством и не глянуть силу тока через падение напряжения на  самом резисторе? Ведь номинал резистора у нас постоянный и почти не изменяется с изменением силы тока 😉

    [quads id=1]

    Осциллограмма силы тока на активном сопротивлении

    В данном опыте нам не обязательно знать номинал силы тока в цепи. Мы будем просто смотреть, от чего зависит сила тока и изменяется ли вообще?

    Поэтому,  наша схема примет вот такой вид:

    В этом случае шунтом будет являться резистор сопротивлением в 0,5 Ом. Почему именно 0,5 Ом? Да потому что он не будет сильно греться, так как обладает маленьким сопротивлением, а также  его номинал вполне достаточен, чтобы снять с него напряжение.

    Осталось снять напряжение с генератора, а также со шунта с помощью осциллографа. Если вы не забыли, со шунта мы снимаем осциллограмму силы тока в цепи. Красная осциллограмма – это напряжение с генератора Uген , а желтая осциллограмма  – это напряжение с шунта Uш , в нашем случае  – сила тока.  Смотрим, что у нас получилось:

    Частота 28 Герц:

    Частота 285 Герц:

    Частота 30 Килогерц:

    Как вы видите, с ростом частоты сила тока у нас осталась такой же.

    Давайте побалуемся формой сигнала:

    Как мы видим, сила тока  полностью повторяет форму сигнала напряжения.

    Итак, какие можно сделать выводы?

    1) Сила тока через активное (омическое) сопротивление имеет такую же форму, как и форма напряжения.

    2) Сила тока и напряжение на активном сопротивлении совпадают по фазе, то есть куда напряжение, туда и ток. Они двигаются синфазно, то есть одновременно.

    3) С ростом частоты ничего не меняется (если только на очень высоких частотах).

    Конденсатор в цепи переменного тока

    Ну а теперь давайте вместо резистора поставим конденсатор.

    Смотрим осциллограммы:

    Как вы видите, конденсатор обладает сопротивлением, так  как сила тока в цепи значительно уменьшилась. Но обратите внимание, что произошел сдвиг желтой осциллограммы, то бишь осциллограммы силы тока.

    Вспоминаем алгебру старшие классы. Итак, полный период T – это

    Теперь давайте прикинем, какой сдвиг фаз у нас получился на графике:

    Где-то примерно П/2 или 90 градусов.

    Почему так произошло? Во всем виновато физическое свойство конденсатора. В самые первые доли секунд, конденсатор ведет себя как проводник с очень малым сопротивлением, поэтому сила тока в этот момент будет максимальна. В этом можно легко убедиться, если резко подать на конденсатор напряжение и в начальный момент времени посмотреть, что происходит с силой тока

    Красная осциллограмма – это напряжение, которое мы подаем на конденсатор, а желтая – это сила тока в цепи конденсатора. По мере заряда конденсатора сила тока падает и достигает нуля при полном заряде конденсатора.

    К чему приведет дальнейшее увеличение частоты? Давайте посмотрим:

    50 Герц.

    100 Герц

    200 Герц

    Как вы видите, с увеличением частоты, у нас сила тока в цепи с конденсатором возрастает.

    Реактивное сопротивление конденсатора

    Как мы увидели с прошлого опыта, с увеличением частоты растет сила тока! Кстати, у резистора не росла. То есть получается в данном случае из закона Ома, что сопротивление конденсатора зависит от частоты! Да, все так оно и есть. Но называется оно не просто сопротивлением, а реактивным сопротивлением и вычисляется по формуле:

    где

    Хс – реактивное сопротивление конденсатора, Ом

    П – постоянная и приблизительно равна 3,14

    F – частота, Гц

    С – емкость конденсатора, Фарад

    Катушка индуктивности в цепи переменного тока

    Ну а теперь давайте возьмем катушку индуктивности вместо конденсатора:

    Проводим все аналогичные операции, как и с конденсатором. Смотрим на осциллограммы в цепи с катушкой индуктивности:

    Если помните, вот такую осциллограмму мы получили в схеме с конденсатором:

    Видите разницу? На катушке индуктивности ток отстает от напряжения на  90 градусов, на П/2, или, как еще говорят, на четверть периода (весь период у нас или 360 градусов).

    Так-так-так…. Давайте соберемся с мыслями. То есть в цепи с переменным синусоидальным током,  ток на конденсаторе опережает напряжение на 90 градусов, а на катушке индуктивности ток отстает от напряжения тоже на 90 градусов? Да, все верно.

    [quads id=1]

    Почему на катушке ток отстает от напряжения?

    Не будем углубляться в различные физические процессы и формулы, просто сочтем за данность, что сила тока не может резко возрастать на катушке индуктивности. Для этого проведем простой опыт. Так же как и на конденсатор, мы резко подадим напряжение на катушку индуктивности, и посмотрим, что случилось с силой тока.

    Как вы видите, при резкой подаче напряжения на катушку, сила тока не стремится также резко возрастать, а возрастает постепенно, если быть точнее, по экспоненте.

    Давайте вспомним, как это было у конденсатора:

    Все с точностью наоборот! Можно даже сказать, что катушка – это полная противоположность конденсатору 😉

    Ну и напоследок давайте еще побалуемся частотой:

    240 Килогерц

    34 Килогерца

    17 Килогерц

    10 Килогерц

    Вывод?

    С уменьшением частоты сила тока через катушку увеличивается.

    Реактивное сопротивление катушки индуктивности

    Из опыта выше мы можем сделать вывод, что сопротивление катушки зависит от частоты и вычисляется по формуле

    где

    ХL –  реактивное сопротивление катушки, Ом

    П – постоянная и приблизительно равна 3,14

    F – частота, Гц

    L – индуктивность, Генри

    Мощность в цепи с реактивными радиоэлементами

    Для дальнейшего объяснения этого явления нам потребуется наша осциллограмма с катушки индуктивности:

    Итак, давайте выделим на ней один период и разделим его на 4 части, то есть по 90 градусов каждая или π/2.

    Давайте начнем с такого понятия, как мощность. Если не забыли, мощность – это сила тока помноженное на напряжение, то есть P=IU. Итак, в первую четвертинку периода t1 у нас напряжение принимает положительные значения и сила тока тоже положительное. Плюс на плюс дает плюс. В эту четверть периода энергия поступает из источника в реактивное сопротивление.

    Теперь давайте рассмотрим отрезок времени t2. Здесь ток со знаком “плюс”, а напряжение со знаком “минус”. В итоге плюс на минус дает минус. Получается мощность со знаком “минус”. А разве так бывает? Еще как бывает! В этот промежуток времени реактивный радиоэлемент отдает запасенную энергию обратно в источник напряжения. Для лучшего понимания давайте рассмотрим простой житейский пример.

    Представим себе кузнеца за работой:

     

    Не знаю, какое было у вас детство, но я когда был пацаном, брал свинец с аккумуляторов и плющил его в металлические пластинки. И что думаете? Свинец нагревался. Не так, чтобы прям обжигал, а был тепленький на ощупь. То есть моя энергия удара превращалась в тепло, можно даже сказать, в полезную энергию.

    А что если взять пружину от стоек ВАЗа и ударять по ней?

    С пружиной не станет НИ-ЧЕ-ГО! Она ведь не свинец. Но… заметьте вот такую вещь: как только мы начинаем “плющить” пружину кувалдой, у нас она начинает сжиматься. И вот она сжалась до упора и… выстрелила вверх, подхватив с собой тяжелую кувалду, которая только что пыталась ее расплющить. То есть в данном случае энергия вернулась обратно в источник энергии, то есть обратно  к кузнецу. Он вроде как и пытался расплющить пружину, но пружина вернула энергию обратно своим разжатием. То есть кузнецу не надо уже было подымать тяжелый молот, так как за него это уже сделала пружина.

    Разжатие пружины и возврат ею энергии обратно – это и есть отрицательная мощность. В этом случае энергия возвращается обратно в источник. Хорошо ли это или плохо – это уже другая история.

    В третий промежуток времени  t3 и ток и напряжение у нас со знаком “минус”. Минус на минус – это плюс. То есть реактивный элемент снова поглощает энергию, ну а на t4, снова ее отдает, так как плюс на минус дает минус.

    В результате за весь период у нас суммарное потребление энергии равно чему?

    Правильно, нулю!

    Так что же это получается тогда? На катушке и конденсаторе не будет выделяться никакой энергии? Получается так. Поэтому в схемах они чаще всего холодные, хотя могут быть и слегка теплыми, так как реальные параметры катушки и конденсатора выглядят совсем по другому.

    Эквивалентная схема реальной катушки индуктивности выглядит вот так:

    где

    R– это сопротивление потерь. Это могут быть потери в проводах, так как любой провод обладает сопротивлением. Это могут быть потери в диэлектрике, потери в сердечнике и потери на вихревые токи.  Как видите, раз есть сопротивление, значит на нем может выделяться мощность, то есть тепло.

    L – собственно сама индуктивность катушки

    С – межвитковая емкость.

    А вот и эквивалентная схема реального конденсатора:

    где

    r – сопротивление диэлектрика и корпуса между обкладками

    С – собственно сама емкость конденсатора

    ESR – эквивалентное последовательное сопротивление

    ESI (ESL) – эквивалентная последовательная индуктивность

    Здесь мы тоже видим такие параметры, как r  и ESR, которые на высоких частотах будут еще лучше себя проявлять, благодаря скин-эффекту. Ну и, соответственно, на них будет выделяться мощность, что приведет к небольшому малозаметному нагреву.

    Резюме

    Резистор обладает активным (омическим) сопротивлением. Катушка индуктивности и конденсатор обладают реактивным сопротивлением.

    В цепи переменного тока на конденсаторе ток опережает напряжение на 90 градусов, а на катушке ток отстает от напряжения на 90 градусов.

    Сопротивление катушки вычисляется по формуле

    Сопротивление конденсатора вычисляется по формуле:

    В цепи переменного тока на идеальном реактивном сопротивлении не выделяется мощность.

    Реальные катушка и конденсатор имеют в своем составе паразитные параметры, которые имеют некоторое сопротивление. Поэтому реальные катушка и конденсатор не обладают чисто реактивным сопротивлением.

    Индуктивное сопротивление: обозначение, сопротивление катушки формула

    Когда в цепи нарастает или уменьшается ток, электромагнитное поле создает противодействующую электродвижущую силу. Это явление порождается индуктивностью катушки. Индуктивное сопротивление воздействует только на переменный ток, быстрые изменения которого порождают противодействующую силу. В статье будет более подробно рассказано о природе этого явления.

    Что зовется индуктивным сопротивлением

    Когда на катушку подают переменное напряжение, ток, проходящий по ней, меняется согласно поданному напряжению. Это служит причиной изменения магнитного поля, создающего электродвижущую силу, препятствующую происходящему.

    Схема для измерения

    В такой цепи имеется зависимость электрических параметров от двух видов: обычного и индуктивного. Они обозначаются, соответственно, как R и XL.

    На обычном происходит выделение мощности. Однако на реактивных элементах она является нулевой. Это связано с постоянным изменением направления переменного тока.

    В течение одного периода колебаний энергия дважды закачивается в катушку и столько же раз возвращается в источник.

    Определение индуктивности

    От каких факторов зависит сопротивление

    Изменение силы тока создает электромагнитное поле переменной интенсивности. Результатом его воздействия на проводник является противодействие происходящему изменению тока.

    Это противодействие называется реактивным сопротивлением. Существуют две его разновидности: индуктивная и емкостная. Первая создается при наличии в схеме индуктивного элемента, вторая — конденсатора.

    В ситуации, когда в цепи присутствует катушка, ее реакция усиливается по мере увеличения частоты.

    Цепь, в которой возникает индукция

    В случае, когда ее индуктивность уменьшается, то противодействующая сила также становится меньше. При увеличении она возрастает.

    Индуктивное сопротивление существенно связано с тем, какую форму принимает проводник. Оно имеется также и у отдельного провода, лежащего прямо. Однако если рядом будет еще один, то он будет оказывать воздействие дополнительно, что повлияет на рассматриваемую величину.

    Рассматриваемую характеристику отдельного провода можно определять в зависимости от его толщины, но оно никак не связано с его сечением.

    Принцип действия электродвижущей силы

    Катушка индуктивности

    Он представляет собой изолированный провод, многократно намотанный вокруг сердечника.

    Обычно каркас имеет цилиндрическую или тороидальную форму.

    Индуктивность рассматривается в качестве основной характеристики катушки. Это качество выражает способность элемента осуществлять преобразование переменного тока в магнитное поле.

    Важно! Магнитные свойства существуют даже у одиночного провода, при условии, что изменяется проходящий через него ток. Воздействие поля направлено так, чтобы противодействовать его изменению. Если он будет увеличиться, поле будет его тормозить, а если ослабевать — усиливать.

    Катушки индуктивности

    Определение направления силовых линий подчиняется «правилу большого пальца»: если у сжатой в кулак руки большой палец указывает в направлении изменения силы тока, то сомкнутые пальцы подсказывают направление силовых линий поля.

    Таким образом в том случае, если провод многократно намотан на цилиндрическое основание, то силовые линии от разных витков складываются и проходят через ось.

    Для того, чтобы многократно увеличить индуктивность, в центр цилиндра помещают сердечник из ферромагнитного материала.

    Индуктивное сопротивление – единицы измерения

    Измерение этой величины производится в омах. Здесь используются такая же единица измерения, как и для резистора, несмотря на то, что у них различная природа. Рассматриваемая величина порождается электродвижущей силой, противодействующей происходящему изменению. Обычное возникает в связи с рассеиванием энергии при прохождении электронов по проводнику.

    Магнитное поле индуктивного элемента

    Индуктивное сопротивление – как его найти

    Реальная катушка имеет не только реактивное, но и обычное сопротивление. Индуктивное сопротивление определяется по формуле:

    XL=2*П*v*L

    Здесь употреблены следующие обозначения:

    1. XL – рассматриваемая величина.
    2. Символом «П» обозначено число Пи.
    3. V представляет собой частоту.
    4. L — это обозначение величины индуктивности.

    Надо отметить, что величина (2*П*v) представляют собой круговую частоту, которую обозначают греческим символом «омега».

    Катушки с различными сердечниками

    Рассматриваемая величина подчиняется закону Ома. Формула выглядит так:

    I = U / XL

    I, U представляют собой ток и напряжение, XL – это индуктивное сопротивление.

    Конфигурация магнитного поля катушки

    Для определения искомой величины можно воспользоваться приведенными формулами. При этом можно воспользоваться амперметром и вольтметром. Первый из них надо включить последовательно, второй — параллельно.

    При этом необходимо учитывать следующее. На самом деле, в цепи, в которую включена индуктивность, действует два вида сопротивления: активное и реактивное. Измерив ток и напряжение, можно определить их результирующую величину. Нужно помнить, что она не является их простой суммой.

    Дело в том, что в переменной цепи, где имеется только катушка и нет конденсатора, напряжение находится впереди тока на четверть периода колебания. Эта величина равна 90 градусам.

    Полное сопротивление определяется следующим образом. Для этого необходимо нарисовать соответствующую диаграмму. Если по горизонтали отложить величину обычного, а по вертикали — реактивного, а затем по этим векторам построить прямоугольник, то длина его диагонали будет равна полному значению.

    Магнитное поле провода

    К примеру, если подобрать элементы цепи таким образом, чтобы по абсолютной величине обе этих величины были равны, то искомая часть определится как их полное значение, умноженное на квадратный корень из двух.

    Для того, чтобы получить информацию о зависимости индуктивного сопротивления от частоты, возможно воспользоваться осциллографом.

    При использовании переменного тока необходимо учитывать не только обычное, но и индуктивное сопротивление. Оно возникает в том случае, если в электрической цепи присутствует катушка.

    Реактивное сопротивление, индуктивное и емкостное | Физика

    Цели обучения

    К концу этого раздела вы сможете:

    • Зависимость напряжения и тока от времени в простых индуктивных, емкостных и резистивных цепях.
    • Рассчитайте индуктивное и емкостное сопротивление.
    • Рассчитывайте ток и / или напряжение в простых индуктивных, емкостных и резистивных цепях.

    Многие цепи также содержат конденсаторы и катушки индуктивности в дополнение к резисторам и источнику переменного напряжения.Мы видели, как конденсаторы и катушки индуктивности реагируют на постоянное напряжение при его включении и выключении. Теперь мы исследуем, как катушки индуктивности и конденсаторы реагируют на синусоидальное переменное напряжение.

    Катушки индуктивности и индуктивное сопротивление

    Предположим, что индуктор подключен непосредственно к источнику переменного напряжения, как показано на рисунке 1. Разумно предположить, что сопротивление пренебрежимо мало, поскольку на практике мы можем сделать сопротивление индуктора настолько малым, что оно окажет незначительное влияние на схему.Также показан график зависимости напряжения и тока от времени.

    Рис. 1. (a) Источник переменного напряжения, включенный последовательно с катушкой индуктивности, имеющей незначительное сопротивление. (б) График зависимости тока и напряжения на катушке индуктивности от времени.

    График на Рисунке 1 (b) начинается с максимального напряжения. Обратите внимание, что ток начинается с нуля и достигает своего пика после напряжения, которое им управляет, как это было в случае, когда напряжение постоянного тока было включено в предыдущем разделе.Когда напряжение становится отрицательным в точке а, ток начинает уменьшаться; оно становится нулевым в точке b, где напряжение является самым отрицательным. Затем ток становится отрицательным, снова вслед за напряжением. Напряжение становится положительным в точке c и начинает делать ток менее отрицательным. В точке d ток проходит через ноль, когда напряжение достигает своего положительного пика, чтобы начать следующий цикл. Кратко это поведение можно описать следующим образом:

    Напряжение переменного тока в индукторе

    Когда на катушку индуктивности подается синусоидальное напряжение, оно опережает ток на одну четверть цикла или на фазовый угол 90º.

    Ток отстает от напряжения, поскольку индукторы препятствуют изменению тока. Изменение тока индуцирует обратную ЭДС В = — L I / Δ t ). Это считается эффективным сопротивлением катушки индуктивности переменному току. Среднеквадратичный ток I через катушку индуктивности L определяется версией закона Ома:

    [латекс] I = \ frac {V} {{X} _ {L}} \\ [/ latex],

    , где В, — среднеквадратичное значение напряжения на катушке индуктивности, а X L определяется как

    .

    [латекс] {X} _ {L} = 2 \ pi {fL} \\ [/ латекс],

    с f частота источника переменного напряжения в герцах (анализ схемы с использованием правила петли Кирхгофа и вычислений фактически дает это выражение). X L называется индуктивным реактивным сопротивлением , потому что катушка индуктивности препятствует прохождению тока. X L имеет единицы измерения Ом (1 Гн = 1 Ом с, так что частота, умноженная на индуктивность, имеет единицы (циклы / с) (Ом ⋅ с) = Ом)), что соответствует его роли в качестве эффективное сопротивление. Имеет смысл, что X L пропорционально L , поскольку чем больше индукция, тем больше его сопротивление изменению.Также разумно, что X L пропорционально частоте f , поскольку большая частота означает большее изменение тока. То есть Δ I / Δ t является большим для больших частот (большие f , маленькие Δ t ). Чем больше изменение, тем больше сопротивление катушки индуктивности.

    Пример 1. Расчет индуктивного сопротивления, а затем тока

    (a) Рассчитайте индуктивное сопротивление 3.Индуктор 00 мГн при подаче переменного напряжения 60,0 Гц и 10,0 кГц. (b) Каков среднеквадратичный ток на каждой частоте, если приложенное действующее напряжение составляет 120 В?

    Стратегия

    Индуктивное реактивное сопротивление находится непосредственно из выражения X L = 2πf L . Как только X L было найдено на каждой частоте, закон Ома, как указано в уравнении I = В / X L , может быть использован для определения тока на каждой частоте.

    Решение для (а)

    Ввод частоты и индуктивности в уравнение X L = 2πf L дает

    X L = 2πf L = 6,28 (60,0 / с) (3,00 мГн) = 1,13 Ом при 60 Гц.

    Аналогично, на 10 кГц,

    X L = 2πf L = 6,28 (1,00 × 10 4 / с) (3,00 мГн) = 188 Ом при 10 кГц.

    Решение для (b)

    Среднеквадратичное значение тока теперь определяется с использованием версии закона Ома в уравнении I = В / X L , при условии, что приложенное действующее напряжение составляет 120 В.Для первой частоты это дает

    [латекс] I = \ frac {V} {{X} _ {L}} = \ frac {120 \ text {V}} {1.13 \ text {} \ Omega} = 106 \ text {A at} 60 \ текст {Hz} \\ [/ latex].

    Аналогично, на 10 кГц,

    [латекс] I = \ frac {V} {{X} _ {L}} = \ frac {120 \ text {V}} {188 \ text {} \ Omega} = 0,637 \ text {A at} 10 \ текст {кГц} \\ [/ latex].

    Обсуждение

    Катушка индуктивности по-разному реагирует на двух разных частотах. На более высокой частоте его реактивное сопротивление велико, а ток невелик, что соответствует тому, как катушка индуктивности препятствует быстрому изменению.Таким образом, наиболее затруднены высокие частоты. Индукторы могут использоваться для фильтрации высоких частот; например, большую катушку индуктивности можно включить последовательно с системой воспроизведения звука или последовательно с вашим домашним компьютером, чтобы уменьшить высокочастотный звук, выводимый из ваших динамиков или высокочастотные всплески мощности на ваш компьютер.

    Обратите внимание, что, хотя сопротивлением в рассматриваемой цепи можно пренебречь, переменный ток не очень велик, потому что индуктивное реактивное сопротивление препятствует его протеканию.С переменным током нет времени, чтобы ток стал слишком большим.

    Конденсаторы и емкостное сопротивление

    Рассмотрим конденсатор, подключенный непосредственно к источнику переменного напряжения, как показано на рисунке 2. Сопротивление такой цепи можно сделать настолько малым, что оно окажет незначительное влияние по сравнению с конденсатором, поэтому мы можем предположить, что сопротивление незначительно. Напряжение на конденсаторе и ток показаны на рисунке как функции времени.

    Рисунок 2.(а) Источник переменного напряжения, включенный последовательно с конденсатором С, имеющим незначительное сопротивление. (б) График зависимости тока и напряжения на конденсаторе от времени.

    График на Рисунке 2 начинается с максимального напряжения на конденсаторе. В этот момент ток равен нулю, потому что конденсатор полностью заряжен и останавливает поток. Затем напряжение падает, а ток становится отрицательным по мере разряда конденсатора. В точке а конденсатор полностью разряжен (на нем Q = 0 ) и напряжение на нем равно нулю.Ток остается отрицательным между точками a и b, вызывая обратное напряжение на конденсаторе. Это завершается в точке b, где ток равен нулю, а напряжение имеет самое отрицательное значение. Ток становится положительным после точки b, нейтрализуя заряд конденсатора и доводя напряжение до нуля в точке c, что позволяет току достичь своего максимума. Между точками c и d ток падает до нуля, когда напряжение достигает своего пика, и процесс начинает повторяться. На протяжении всего цикла напряжение соответствует тому, что делает ток, на четверть цикла:

    Напряжение переменного тока в конденсаторе

    Когда на конденсатор подается синусоидальное напряжение, оно следует за током на одну четверть цикла или на фазовый угол 90º.

    Конденсатор влияет на ток, имея возможность полностью его отключить, когда он полностью заряжен. Поскольку применяется переменное напряжение, возникает среднеквадратичный ток, но он ограничивается конденсатором. Это считается эффективным сопротивлением конденсатора переменному току, поэтому среднеквадратичный ток I в цепи, содержащей только конденсатор C , определяется другой версией закона Ома как

    .

    [латекс] I = \ frac {V} {{X} _ {C}} \\ [/ latex],

    , где В, — среднеквадратичное значение напряжения, а X C определено (Как и в случае с X L , это выражение для X C является результатом анализа цепи используя правила и исчисление Кирхгофа) равным

    [латекс] {X} _ {C} = \ frac {1} {2 \ pi fC} \\ [/ latex],

    , где X C называется емкостным реактивным сопротивлением , потому что конденсатор препятствует прохождению тока. X C имеет единицы измерения Ом (проверка оставлена ​​в качестве упражнения для читателя). X C обратно пропорциональна емкости C ; Чем больше конденсатор, тем больший заряд он может накапливать и тем больше может протекать ток. Она также обратно пропорциональна частоте f ; чем выше частота, тем меньше времени остается для полной зарядки конденсатора и, следовательно, меньше препятствует току.

    Пример 2. Расчет емкостного реактивного сопротивления, а затем тока

    (a) Рассчитайте емкостное реактивное сопротивление конденсатора 5,00 мФ при приложении переменного напряжения 60,0 Гц и 10,0 кГц. (b) Каков среднеквадратичный ток, если приложенное действующее напряжение составляет 120 В?

    Стратегия

    Емкостное реактивное сопротивление находится непосредственно из выражения в [latex] {X} _ {C} = \ frac {1} {2 \ pi fC} \\ [/ latex]. После того, как X C было обнаружено на каждой частоте, закон Ома, сформулированный как I = В, / X C , можно использовать для определения тока на каждой частоте.

    Решение для (а)

    Ввод частоты и емкости в [латекс] {X} _ {C} = \ frac {1} {2 \ pi fC} \\ [/ latex] дает

    [латекс] \ begin {array} {lll} {X} _ {C} & = & \ frac {1} {2 \ pi fC} \\ & = & \ frac {1} {6.28 \ left (60.0 / \ text {s} \ right) \ left (5.00 \ text {} \ mu \ text {F} \ right)} = 531 \ text {} \ Omega \ text {at} 60 \ text {Hz} \ end {массив }\\[/латекс].

    Аналогично, на 10 кГц,

    [латекс] \ begin {array} {lll} {X} _ {C} & = & \ frac {1} {2 \ pi fC} = \ frac {1} {6.{4} / \ text {s} \ right) \ left (5,00 \ mu \ text {F} \ right)} \\ & = & 3,18 \ text {} \ Omega \ text {at} 10 \ text {кГц} \ end {array} \\ [/ latex].

    Решение для (b)

    Среднеквадратичное значение тока теперь определяется с использованием версии закона Ома в I = В / X C , учитывая приложенное действующее напряжение 120 В. Для первой частоты это дает

    [латекс] I = \ frac {V} {{X} _ {C}} = \ frac {120 \ text {V}} {531 \ text {} \ Omega} = 0,226 \ text {A at} 60 \ текст {Hz} \\ [/ latex].

    Аналогично, на 10 кГц,

    [латекс] I = \ frac {V} {{X} _ {C}} = \ frac {120 \ text {V}} {3.18 \ text {} \ Omega} = 3.37 \ text {A at} 10 \ текст {Hz} \\ [/ latex].

    Обсуждение

    Конденсатор очень по-разному реагирует на двух разных частотах, а индуктор реагирует прямо противоположным образом. На более высокой частоте его реактивное сопротивление мало, а ток велик. Конденсаторы одобряют изменения, а индукторы — противодействуют. Конденсаторы больше всего препятствуют низким частотам, так как низкая частота позволяет им успеть зарядиться и остановить ток.Конденсаторы можно использовать для фильтрации низких частот. Например, конденсатор, включенный последовательно с системой воспроизведения звука, избавляет ее от гула 60 Гц.

    Хотя конденсатор в основном представляет собой разомкнутую цепь, в цепи с напряжением переменного тока, приложенным к конденсатору, присутствует среднеквадратичный ток. Это связано с тем, что напряжение постоянно меняет направление, заряжая и разряжая конденсатор. Если частота стремится к нулю (постоянный ток), X C стремится к бесконечности, и ток равен нулю, когда конденсатор заряжен.На очень высоких частотах реактивное сопротивление конденсатора стремится к нулю — он имеет незначительное реактивное сопротивление и не препятствует току (он действует как простой провод). Конденсаторы оказывают противоположное влияние на цепи переменного тока, чем индукторы .

    Резисторы в цепи переменного тока

    В качестве напоминания рассмотрим Рисунок 3, на котором показано напряжение переменного тока, приложенное к резистору, и график зависимости напряжения и тока от времени. Напряжение и ток равны точно в фазе в резисторе.Отсутствует частотная зависимость поведения простого сопротивления в цепи:

    Рис. 3. (a) Источник переменного напряжения, включенный последовательно с резистором. (b) График зависимости тока и напряжения на резисторе от времени, показывающий, что они точно совпадают по фазе.

    Напряжение переменного тока в резисторе

    Когда на резистор подается синусоидальное напряжение, напряжение точно совпадает по фазе с током — они имеют фазовый угол 0 °.

    Сводка раздела

    • Для катушек индуктивности в цепях переменного тока мы обнаруживаем, что когда на катушку подается синусоидальное напряжение, напряжение опережает ток на одну четверть цикла или на фазовый угол 90 °.
    • Противодействие катушки индуктивности изменению тока выражается как сопротивление переменному току.
    • Закон Ома для катушки индуктивности

      [латекс] I = \ frac {V} {{X} _ {L}} \\ [/ latex],

      , где В, — среднеквадратичное значение напряжения на катушке индуктивности.

    • X L определяется как индуктивное реактивное сопротивление, определяемое по формуле

      [латекс] {X} _ {L} = 2 \ pi fL \\ [/ латекс],

      с f частота источника переменного напряжения в герцах.

    • Индуктивное реактивное сопротивление X L выражается в единицах Ом и имеет наибольшее значение на высоких частотах.
    • Для конденсаторов мы обнаруживаем, что когда на конденсатор подается синусоидальное напряжение, напряжение следует за током на одну четверть цикла или на фазовый угол 90º.
    • Поскольку конденсатор может останавливать ток при полной зарядке, он ограничивает ток и предлагает другую форму сопротивления переменному току; Закон Ома для конденсатора

      [латекс] I = \ frac {V} {{X} _ {C}} \\ [/ latex],

      , где В, — среднеквадратичное значение напряжения на конденсаторе.

    • X C определяется как емкостное реактивное сопротивление, определяемое по формуле

      [латекс] {X} _ {C} = \ frac {1} {2 \ pi fC} \\ [/ latex].

    • X C имеет единицы измерения Ом и имеет наибольшее значение на низких частотах.

    Концептуальные вопросы

    1. Пресбиакузис — это возрастная потеря слуха, которая постепенно влияет на высокие частоты. Усилитель слухового аппарата предназначен для равномерного усиления всех частот. Чтобы отрегулировать его мощность на пресбиакузис, включите ли вы конденсатор последовательно или параллельно динамику слухового аппарата? Объяснять.

    2. Будете ли вы использовать большую индуктивность или большую емкость последовательно с системой для фильтрации низких частот, таких как гул 100 Гц в звуковой системе? Объяснять.

    3. Высокочастотный шум в сети переменного тока может повредить компьютеры. Использует ли съемный блок, предназначенный для предотвращения этого повреждения, большую индуктивность или большую емкость (последовательно с компьютером) для фильтрации таких высоких частот? Объяснять.

    4. Зависит ли индуктивность от тока, частоты или и того, и другого? А как насчет индуктивного сопротивления?

    5. Объясните, почему конденсатор на рисунке 4 (а) действует как фильтр низких частот между двумя цепями, тогда как конденсатор на рисунке 4 (b) действует как фильтр высоких частот.

    Рисунок 4. Конденсаторы и катушки индуктивности. Конденсатор с высокой и низкой частотой.

    6. Если конденсаторы на Рисунке 4 заменить катушками индуктивности, что будет действовать как фильтр низких частот, а какой — как фильтр высоких частот?

    Задачи и упражнения

    1. На какой частоте индуктор 30,0 мГн будет иметь реактивное сопротивление 100 Ом?

    2. Какое значение индуктивности следует использовать, если требуется реактивное сопротивление 20,0 кОм при частоте 500 Гц?

    3.Какую емкость следует использовать для получения реактивного сопротивления 2,00 МОм при 60,0 Гц?

    4. На какой частоте конденсатор 80,0 мФ будет иметь реактивное сопротивление 0,250 Ом?

    5. (a) Найдите ток через катушку индуктивности 0,500 H, подключенную к источнику переменного тока 60,0 Гц, 480 В. (б) Каким будет ток на частоте 100 кГц?

    6. (a) Какой ток течет, когда источник переменного тока 60,0 Гц, 480 В подключен к конденсатору 0,250 мкФ? (b) Каким будет ток на частоте 25,0 кГц?

    7. А 20.Источник 0 кГц, 16,0 В, подключенный к катушке индуктивности, вырабатывает ток 2,00 А. Что такое индуктивность?

    8. Источник 20,0 Гц, 16,0 В вырабатывает ток 2,00 мА при подключении к конденсатору. Какая емкость?

    9. (a) Катушка индуктивности, предназначенная для фильтрации высокочастотного шума от источника питания, подаваемого на персональный компьютер, включается последовательно с компьютером. Какая минимальная индуктивность должна обеспечивать реактивное сопротивление 2,00 кОм для шума 15,0 кГц? (б) Каково его реактивное сопротивление при 60?0 Гц?

    10. Конденсатор на рисунке 4 (а) предназначен для фильтрации низкочастотных сигналов, препятствуя их передаче между цепями. (а) Какая емкость необходима для создания реактивного сопротивления 100 кОм при частоте 120 Гц? (б) Каким было бы его реактивное сопротивление на частоте 1,00 МГц? (c) Обсудите значение ваших ответов на (a) и (b).

    11. Конденсатор на Рисунке 4 (b) будет фильтровать высокочастотные сигналы, замыкая их на землю / землю. (a) Какая емкость необходима для получения реактивного сопротивления [латекса] \ text {10.0 м \ Omega} [/ latex] для сигнала 5,00 кГц? (б) Каким будет его реактивное сопротивление при 3,00 Гц? (c) Обсудите значение ваших ответов на (a) и (b).

    12. Необоснованные результаты При регистрации напряжений, обусловленных мозговой активностью (ЭЭГ), сигнал 10,0 мВ с частотой 0,500 Гц подается на конденсатор, создавая ток 100 мА. Сопротивление незначительное. а) Какая емкость? б) Что неразумного в этом результате? (c) Какое предположение или предпосылка ответственны?

    13. Создайте свою проблему Рассмотрите возможность использования индуктора последовательно с компьютером, работающим от электричества 60 Гц. Постройте задачу, в которой вы вычисляете относительное снижение напряжения входящего высокочастотного шума по сравнению с напряжением 60 Гц. Среди вещей, которые следует учитывать, — допустимое последовательное реактивное сопротивление катушки индуктивности для мощности 60 Гц и вероятные частоты шума, проходящего через линии электропередач.

    Глоссарий

    индуктивное реактивное сопротивление:
    противодействие катушки индуктивности изменению тока; рассчитывается по X L = 2π fL
    емкостное реактивное сопротивление:
    сопротивление конденсатора изменению тока; рассчитывается по [latex] {X} _ {C} = \ frac {1} {2 \ pi fC} \\ [/ latex]

    Избранные решения проблем и упражнения

    1.531 Гц

    3. 1,33 нФ

    5. (а) 2,55 А (б) 1,53 мА

    7. 63,7 мкГн

    9. (а) 21,2 мГн (б) 8,00 Ом

    В чем разница между сопротивлением, реактивным сопротивлением и импедансом?

    Цепи постоянного тока относительно легко анализировать, поскольку ток течет в одном направлении, а сопротивление является основным элементом цепи. Цепи переменного тока, с другой стороны, более сложны, поскольку напряжение и ток меняют направление с заданной частотой.В то время как цепи постоянного тока имеют сопротивление, цепи переменного тока часто имеют сопротивление и другое свойство, известное как реактивное сопротивление. Импеданс — это комбинация сопротивления и реактивного сопротивления.


    Компоненты, известные как резисторы, предотвращают протекание тока — другими словами, они обладают свойством сопротивления. Резисторы присутствуют как в цепях переменного, так и в постоянном токе, и энергия, которая не может течь, отводится в виде тепла. Математически сопротивление — это просто напряжение, разделенное на ток.

    R = сопротивление (Ом)

    В = напряжение (вольт)

    I = ток (амперы)


    Реактивное сопротивление — это свойство, которое противодействует изменению тока и присутствует как в индукторах, так и в конденсаторах. Поскольку оно влияет только на , изменяя ток , реактивное сопротивление зависит от мощности переменного тока и зависит от частоты тока. Когда присутствует реактивное сопротивление, оно создает фазовый сдвиг на 90 градусов между напряжением и током, причем направление сдвига зависит от того, является ли компонент катушкой индуктивности или конденсатором.

    Реактивное сопротивление, возникающее в катушке индуктивности, известно как индуктивное реактивное сопротивление . Когда присутствует индуктивное реактивное сопротивление, энергия накапливается в виде изменяющегося магнитного поля, и форма волны тока отстает от формы волны напряжения на 90 градусов. Индуктивное реактивное сопротивление возникает из-за устройств, в которых провод намотан по кругу, например катушек (включая линейные реакторы), дросселей и трансформаторов.

    X L = индуктивное реактивное сопротивление (Ом)

    f = частота (Гц)

    L = индуктивность (Генри)

    Реактивное сопротивление конденсатора известно как емкостное реактивное сопротивление .Емкостное реактивное сопротивление накапливает энергию в виде изменяющегося электрического поля и вызывает ток в отведении от напряжения на 90 градусов. Емкость создается, когда две проводящие пластины размещаются параллельно друг другу с небольшим расстоянием между ними, заполненные диэлектрическим материалом (изолятором).

    X C = емкостное реактивное сопротивление (Ом)

    C = емкость (фарады)


    Импеданс — это комбинация сопротивления и реактивного сопротивления (как индуктивного, так и емкостного). — комплексное число, содержащее как действительную, так и мнимую части.(Реальная часть импеданса — это сопротивление, а мнимая часть — реактивное сопротивление.) Импеданс имеет как величину, так и фазу.

    Z = величина импеданса (Ом) в последовательной цепи

    X T = полное реактивное сопротивление (Ом) = X L — X C

    θ = фаза полного сопротивления (градусы)

    Формула вывода и векторная диаграмма

    Катушка индуктивности обычно представляет собой катушку с проволокой.Когда через эту катушку проходит ток, создается электрическое поле. Таким образом, поле будет индуцировано. Наведенное поле зависит от количества витков, а это индуктивность. Катушка индуктивности из-за своей индуктивности сопротивляется протеканию переменного тока. Согласно закону Ленца, индуктор будет сопротивляться изменению тока. Реактивное сопротивление определяется как сопротивление, оказываемое переменным токам только конденсаторами и катушками индуктивности. Обычно реактивное сопротивление обозначается буквой X, частота — f, индуктивность — L, а емкость — C.Индуктивное реактивное сопротивление зависит от частоты. Расчет этого можно сделать с помощью простых формул.

    Что такое индуктивное реактивное сопротивление?

    Индуктивное реактивное сопротивление определяется как эффект, благодаря которому уменьшается протекание переменного тока в катушке индуктивности. Любой переменный ток или изменяющийся ток будут затруднены из-за связанной с ним индуктивности.

    Дроссели и индукторы — это в основном петли из проволоки или катушек, которые наматываются на некоторый ферромагнитный материал или вокруг полого трубчатого формирователя для увеличения их индуктивного значения, называемого индуктивностью.Когда напряжение подается на клеммы индуктора, индукторы накапливают энергию в виде магнитного поля. Напряжение обратной ЭДС катушки индуктивности пропорционально скорости изменения тока, протекающего через нее.

    Индуктивное реактивное сопротивление

    Если мы начали исследовать или наблюдать самоиндуктивность и эффекты, возникающие в цепи, мы легко можем узнать причину этого индуктивного реактивного сопротивления. Чтобы определить это в простой форме, это похоже на противостояние текущему потоку.

    Когда на катушку индуктивности подается напряжение переменного тока, протекание тока будет отличаться от того, когда на нее подается напряжение постоянного тока. Разность фаз между сигналами напряжения и тока создается синусоидальным источником питания. Как частота формы волны, так и индуктивность катушки определяют противодействие протеканию тока в обмотках катушки в цепи переменного тока.

    Сопротивление переменного тока определяет противодействие протеканию тока через катушку в цепи переменного тока и обычно известно как полное сопротивление (Z) цепи.Чтобы отличить сопротивление постоянному току от сопротивления переменного тока, сопротивление всегда ассоциируется с цепями постоянного тока, и общий термин, используемый для этого, — реактивное сопротивление. Значение реактивного сопротивления измеряется в омах, как и сопротивление. Чтобы узнать разницу между значением реактивного сопротивления и значением сопротивления, значение реактивного сопротивления обозначается символом «X».

    Поскольку мы фокусируемся на компонентной катушке индуктивности, реактивное сопротивление катушки индуктивности называется индуктивным реактивным сопротивлением. Чтобы упростить его в другой форме, его можно определить как электрическое сопротивление индуктора при использовании в цепи переменного тока.Обозначение индуктивного реактивного сопротивления — XL.

    Формула индуктивного реактивного сопротивления

    Индуктивное реактивное сопротивление можно рассчитать по следующей формуле. Индуктивное реактивное сопротивление — это произведение удвоенного числа пи, индуктивности катушки и частоты переменного тока. Таким образом, формула может быть обозначена как

    XL = 2πfL

    Где XL — индуктивное реактивное сопротивление, измеряемое в омах

    f — частота переменного тока источника питания в герцах

    L — индуктивность стоимость катушки в генри.

    Из этой формулы индуктивного реактивного сопротивления мы можем четко знать, что индуктивное реактивное сопротивление будет увеличиваться при увеличении частоты или индуктивности. Индуктивное реактивное сопротивление достигает бесконечности, когда частота приближается к бесконечности, и это действует как разомкнутая цепь. Точно так же он приближается к нулю, когда частота уменьшается до нуля, и это действует как короткое замыкание. он четко определяет, что частота и индуктивное сопротивление прямо пропорциональны.

    Идеальный индуктор — это индуктор, который имеет только индуктивность, но не имеет сопротивления и емкости.Хотя идеальных катушек индуктивности не существует. Чтобы определить формулы и расчеты, представим себе размещение идеального индуктора. Если к идеальной катушке индуктивности приложена синусоида, реактивное сопротивление препятствует прохождению тока и подчиняется закону сопротивления

    XL = V / I

    «XL» — индуктивное реактивное сопротивление в омах

    «I» — ток в амперах

    «V» — это напряжение в вольтах.

    В зависимости индуктивного сопротивления от частоты, поскольку мы четко знаем, что частота прямо пропорциональна частоте.Таким образом, мы можем видеть, что катушка индуктивности имеет нулевое реактивное сопротивление при постоянном токе, а катушка индуктивности имеет бесконечное реактивное сопротивление на высоких частотах.

    Выведение формулы индуктивного реактивного сопротивления

    Синусоидальное переменное

    Напряжение V = Vmsinwt

    Обратная ЭДС индукции, e = Ldi / dt

    e = v

    Ldi / dt = Vmsinwt

    Diwtd

    Diwtd. с обеих сторон

    ∫ di = ∫Vm / L sinwtdt

    I = Vm / wL sin (wt-π / 2)

    При wt = π sin (wt-π / 2) = 1

    Im = Vm / wL

    Vm / Im = wL

    W = 2πf

    Индуктивное реактивное сопротивление XL = Vm / Im = 2πfL

    Подача переменного тока через последовательную цепь LR

    В основном мы рассматриваем чисто индуктивную катушку, но каждый раз, рассматривая чисто индуктивную катушку, Это невозможно, но у соленоидов будет определенное сопротивление, неважно, насколько маленьким оно было связано с витками катушки используемого провода.Так что мы можем рассматривать эту простую катушку, поскольку это сопротивление последовательно с индуктивностью.

    Сопротивление «R» и индуктивность «L» присутствуют в цепи переменного тока, а напряжение V будет векторной суммой VL и VR. Фазовый угол цепи определяется новым фазовым углом между током и напряжением и дается греческим символом фи.

    Векторная диаграмма результирующего напряжения

    Здесь линия OB — это опорный горизонтальный ток, OA — это напряжение на резистивной составляющей, находящееся в фазе с током, OC представляет собой индуктивное напряжение, которое на 90 градусов впереди тока, OD даст результирующее напряжение питания.

    Векторная диаграмма результирующего напряжения

    V = действующее значение приложенного напряжения

    I = действующее значение последовательного тока

    VR = падение напряжения IR на сопротивлении, которое находится в фазе с током

    VL = падение напряжения на индуктивности который ведет ток под углом 90 градусов

    Фазовый угол

    Когда две или более индуктивных катушки соединены последовательно, в противном случае одна катушка при последовательном соединении со многими неиндуктивными сопротивлениями, общее сопротивление резистивных элементов будет равно R1 + R2 + R3 , так далее.это даст общее значение сопротивления для цепи.

    Фазовый угол

    Что касается реактивного сопротивления, полное реактивное сопротивление индуктивных элементов равно X1 + X2 + X3 и т. Д., Что дает общее значение реактивного сопротивления цепи.

    Разница между индуктивностью и реактивным сопротивлением включает

    • Реактивное сопротивление — это влияние индуктивности на заданной частоте.
    • Индуктивность определяется как физическое свойство проводника или катушки. Единицы измерения — генри, и это не зависит от частоты сигнала в компоненте.
    • Что касается реактивного сопротивления, оно зависит от частоты сигнала.

    Это полное описание индуктивного реактивного сопротивления, его формула, единицы измерения и размеры. Вот вопрос, в чем разница между реактивным сопротивлением и емкостью.

    Индуктивное сопротивление

    • Изучив этот раздел, вы сможете описать:
    • • Индуктивное реактивное сопротивление.
    • • Соотношение между реактивным сопротивлением, частотой и индуктивностью.
    • • Графическое представление индуктивного сопротивления.

    Когда ток в катушке индуктивности изменяется, создается обратная ЭДС, которая противодействует изменению тока, и чем быстрее начальное изменение тока, тем больше обратная ЭДС. Поэтому неудивительно, что более высокие скорости изменения тока, которые происходят при увеличении частоты волны, вызывают больший эффект обратной ЭДС, который, в свою очередь, уменьшает ток в большей степени, чем на более низких частотах.

    Это переменное сопротивление току, протекающему в катушке индуктивности, связано с величиной индуктивности, потому что чем больше значение индуктивности, тем больше возникает эффект обратной ЭДС. Противодействие протеканию тока через катушку индуктивности пропорционально величине индуктивности и частоте тока в катушке индуктивности. Это противодействие протеканию тока называется ИНДУКТИВНОЙ РЕАКТИВНОСТЬЮ (X L ). Формула для индуктивного реактивного сопротивления умножает угловую скорость волны переменного тока на значение индуктивности:

    Рис 6.1.1 Индуктивное реактивное сопротивление

    Где 2πƒ или ω — угловая скорость, а L — индуктивность в генри.

    Как и сопротивление, реактивное сопротивление измеряется в омах, но отдельно от сопротивления току, вызванного любым внутренним сопротивлением внутри катушки индуктивности. Большие значения индуктивности (встречающиеся в больших типах катушек индуктивности, используемых на низких частотах) имеют более высокие значения внутреннего сопротивления, чем гораздо меньшие типы катушек индуктивности, используемые на радиочастотах и ​​выше. Индукторы — это в основном катушки с проволокой, и чем больше катушек у катушки индуктивности, тем длиннее будет провод и тем больше будет его сопротивление.Это внутреннее сопротивление не может быть отделено от индуктора и должно учитываться при расчетах, особенно в низкочастотных приложениях, где используются большие индукторы. Однако небольшое сопротивление, присутствующее в гораздо меньших радиочастотных индукторах, обычно можно игнорировать.

    На рис. 6.1.1 показан график зависимости индуктивного реактивного сопротивления от частоты для конкретного значения индуктивности, причем X L линейно увеличивается с частотой

    Сопротивление в индукторах

    Сопротивление, присутствующее в проводе больших катушек индуктивности, оказывает заметное влияние на ток и напряжение на катушке индуктивности.Хотя влияние реактивного сопротивления можно рассчитать, оно не будет учитывать общее влияние на ток и напряжение, сопротивление также должно быть принято во внимание. Внутреннее сопротивление индуктора не может быть физически отделено от индуктора, как показано на рис. 6.1.2

    Рис. 6.1.2 Как V

    r и X L влияют на V L на фазорной диаграмме.

    На рис. 6.1.2 также показано влияние внутреннего сопротивления индуктора на его векторную диаграмму.Напряжение на внутреннем сопротивлении (V r ) может быть небольшим по сравнению с напряжением на индуктивности, но V r будет находиться в фазе с эталонным вектором (ток I) и, таким образом, вызовет фазовый сдвиг, вызывающий вектор для V L сместиться в сторону 0 °.

    Поскольку V L представляет собой векторную сумму напряжений V XL и V r (из-за реактивного сопротивления и внутреннего сопротивления катушки индуктивности), она также будет немного больше, чем напряжение (V XL ), которая будет рассчитана только за счет индуктивности.Это означает, что в практической катушке индуктивности вектор напряжения не будет опережать вектор тока точно на + 90 °, фактическая величина фазового сдвига также будет зависеть от величины внутреннего сопротивления. Хотя это не большая проблема с небольшими катушками индуктивности, используемыми в высокочастотных приложениях, это необходимо учитывать в больших, низкочастотных индукторах, где катушка имеет намного больше витков и, следовательно, ее внутреннее сопротивление больше.

    Индуктивное реактивное сопротивление в цепи переменного тока

    Индуктивное реактивное сопротивление

    Переменный ток в катушке индуктивности зависит от приложенного напряжения и индуктивного реактивного сопротивления катушки индуктивности.Индуктивное реактивное сопротивление пропорционально значению индуктивности и частоте переменного напряжения питания.

    Когда переменное напряжение подается на чистую индуктивность, ток, протекающий через индуктивность, отстает от напряжения на клеммах на 90 o .

    Когда через катушку индуктивности протекает изменяющийся ток, возникает самоиндуцированное напряжение. Его полярность такова, что он противостоит изменению. ЭДС напрямую зависит от скорости изменения тока.Это сопротивление называется индуктивным реактивным сопротивлением. И это свойство всех индукторов. Индуктивное реактивное сопротивление измеряется в омах и представлено как X L , где X означает реактивное сопротивление, а нижний индекс L означает, что оно индуктивное. Если изменение тока синусоидальное, индуктивное реактивное сопротивление можно рассчитать по следующему уравнению:

    $ {{X} _ {L}} = 2 \ pi fL \ text {} \ cdots \ text {(1)} $

    Где

    2π = один цикл в радианах

    F = частота в герцах

    L = индуктивность в Генри

    Поскольку угловая скорость, 2πf, часто записывается как ω, мы можем записать уравнение 1 следующим образом:

    $ {{X} _ {L}} = \ omega L \ text {} \ cdots \ text {(2)} $

    Анализ уравнения 1 показывает, что индуктивное сопротивление прямо пропорционально частоте и индуктивности.В следующем примере исследуется эта взаимосвязь:

    Пример

    Рассчитать индуктивное реактивное сопротивление дросселя фильтра 8 Н при 60 Гц и при 120 Гц?

    Решение

    При 60 Гц:

    $ {{X} _ {L}} = 2 \ pi fL = 6,28 * 60 * 8 \ cong 3016 \ Omega $

    При 120 Гц:

    $ { {X} _ {L}} = 2 \ pi fL = 6.28 * 120 * 8 \ cong 6032 \ Omega $

    Этот пример показывает, что если частота удваивается, реактивное сопротивление удваивается таким же образом.

    Если известно реактивное сопротивление определенной индуктивности на данной частоте, ее индуктивность можно определить, переписав уравнение 1 следующим образом:

    $ L = \ frac {{{X} _ {L}}} {2 \ pi } $

    Пример

    Маленькая катушка индуктивности, используемая в ТВ-приемнике, имеет реактивное сопротивление 5 кОм при 45 МГц.{-5}} = 17,69 \ mu H $

    График X L

    Уравнение 1 является линейным, потому что X L изменяется прямо как произведение линейных членов f и L., если мы построим индуктивное реактивное сопротивление в зависимости от частоты, у нас будет график, подобный тому, который показан сплошной линией на рисунке 1, который относится к дросселю фильтра 8 Н в примере 1. Если частота будет увеличена до 120 Гц, линия реактивного сопротивления вырастет за установленные пределы. показано.

    Фиг.1: График зависимости X L от частоты для катушек индуктивности 8 H и 4 H

    Предположим, что индуктивность, использованная в примере 1, была изменена на 4 H, как реактивное сопротивление будет изменяться в зависимости от частоты? Поскольку существует прямая зависимость между индуктивным реактивным сопротивлением и частотой, можно ожидать, что X L уменьшится вдвое от своего значения при 60 Гц и 120 Гц. Построение реактивного сопротивления катушки индуктивности 4 Н на графике на рис. 1 показывает пунктирную линию. Обратите внимание, что наклон (крутизна) вдвое меньше, чем у линии индуктивности 8 Н.Если бы значение L было увеличено до 16 H, можно было бы ожидать, что линия наклона его линии будет вдвое больше, чем у линии 8 H.

    X L последовательно и параллельно

    Индуктивные реактивные сопротивления, включенные последовательно или параллельно, рассматриваются так же, как и сопротивления, как показано на рисунке 2.

    Рисунок 2: Реактивные сопротивления: (a) Последовательно (b) Параллельно

    Следовательно, полное индуктивное сопротивление двух или более последовательно соединенных катушек индуктивности выглядит следующим образом:

    $ {{X} _ {LT}} = {{X} _ {{{L} _ {I}}}} + {{X} _ {{{L} _ {2}}}} + \ cdots + {{X} _ {{{L} _ {n}}}} $

    для два параллельно соединенных индуктивного сопротивления, формула имеет следующий вид:

    $ {{X} _ {LT}} = \ frac {{{X} _ {{{L} _ {I}}}}} {{X} _ {{{L} _ {2}}}}} {{{X} _ {{{L} _ {I}}}} + {{X} _ {{{L} _ {2}}}} } $

    Когда три или более индуктивных сопротивления соединены параллельно, используется обратная формула:

    $ \ frac {1} {{{X} _ {LT}}} = \ frac {1} {{{X} _ {{{L} _ {1}}}}} + \ frac {1} {{{X} _ {{{L} _ {2}}}}} + \ frac {1} {{{X} _ {{{L} _ {3}}}}} + \ cdots + \ frac {1} {{{X} _ {{{L} _ {n}}}}} $

    Падения напряжения и протекание тока рассчитываются с использованием ля Ома w, за исключением того, что X L заменяет R.

    Индуктивная восприимчивость

    Сопротивление, обратное сопротивлению R, является проводимостью G, которая является мерой способности резистивной цепи пропускать ток. Точно так же индуктивное реактивное сопротивление X L имеет обратную индуктивную сопротивляемость, для которой обозначено обозначение B L .

    Индуктивная проводимость — это мера способности чисто индуктивной цепи пропускать ток, и, как и проводимость, ее единицей является Siemens, S.

    Формула индуктивной проводимости может быть выражена как:

    $ {{B} _ {L}} = \ frac {1} {{{X} _ {L}}} $

    Объяснение урока: Резонанс в цепях переменного тока

    В этом пояснителе мы узнаем, как рассчитать резонансную частоту простого резистивного -емкостно-индуктивные цепи.

    Цепь, содержащая резистор (R), катушку индуктивности (L) и конденсатор (C), подключенную к источнику переменного разности потенциалов. показано на следующем рисунке.

    Источник переменной разности потенциалов чередует определенную частоту.

    На полное сопротивление цепи влияет частота переменной разности потенциалов. Увеличение частоты переменной разности потенциалов увеличивает индуктивное реактивное сопротивление и уменьшает емкостное реактивное сопротивление (но не в равной степени).Импеданс цепи зависит от абсолютной разницы между индуктивным и емкостным реактивным сопротивлением цепи.

    Полное сопротивление последовательной цепи переменного тока с индуктивностью и емкостью определяется по формуле.

    Формула: Импеданс

    Импеданс цепи определяется выражением 𝑍 = 𝑅 + (𝑋 − 𝑋) ,  где 𝑅 — сопротивление цепи, 𝑋 — индуктивное сопротивление цепи, и 𝑋 — емкостное реактивное сопротивление цепи.

    Резонансная частота цепи — это частота приложенной переменной разности потенциалов, которая создает наибольшую ток в цепи.

    Переменная частота разности потенциалов вблизи резонансной частоты генерирует токи, близкие к величине резонансной. частотный ток, тогда как частоты дальше от резонансной частоты генерируют токи меньшей величины.

    На следующем рисунке показано, как максимальный ток в цепи зависит от частоты.

    Из формулы для импеданса, 𝑍 = 𝑅 + (𝑋 − 𝑋) ,  мы можем видеть это когда 𝑋 − 𝑋 = 0,  это должно быть так, что 𝑍 = 𝑅, что соответствует минимальному сопротивлению цепи.Ток наибольший для этого импеданса.

    Значение емкостного реактивного сопротивления определяется формулой.

    Формула: емкостное реактивное сопротивление

    Емкостное реактивное сопротивление 𝑋 цепи с емкостью, несущей переменный ток с частотой определяется выражением 𝑋 = 12𝜋𝑓𝐶.

    Значение индуктивного реактивного сопротивления определяется формулой.

    Формула: индуктивное реактивное сопротивление

    Индуктивное реактивное сопротивление цепи с индуктивностью, несущей переменный ток. ток с частотой определяется выражением 𝑋 = 2𝜋𝑓𝐿.

    Для минимального импеданса необходимо, чтобы 2𝜋𝑓𝐿 − 12𝜋𝑓𝐶 = 0.

    Это уравнение можно переформулировать следующим образом: 2𝜋𝑓𝐿 = 12𝜋𝑓𝐶2𝜋𝑓 = 12𝜋𝑓𝐿𝐶 (2𝜋𝑓) = 1𝐿𝐶2𝜋𝑓 = 1𝐿𝐶,  где 𝑓 — резонансная частота контура.

    Отсюда получаем формулу для резонансной частоты.

    Формула: резонансная частота

    Резонансная частота цепи с индуктивностью и емкость 𝐶 определяется выражением 2𝜋𝑓 = 1𝐿𝐶.

    Давайте теперь рассмотрим несколько примеров.

    Пример 1: Определение резонансной частоты контура

    Какова резонансная частота контура, показанного на схеме? Дайте ответ с точностью до одного десятичного знака.

    Ответ

    Резонансная частота определяется выражением 2𝜋𝑓 = 1𝐿𝐶.

    Подставляя значения, указанные в вопросе, получаем 2𝜋𝑓 = 17,5 × 3,5 × 10. HF

    С точностью до одного десятичного знака 𝑓 равно 3,1 Гц.

    Пример 2: Определение пикового тока в цепи, колеблющейся на своей резонансной частоте

    Цепь, содержащая последовательно включенные резистор, конденсатор и катушку индуктивности, имеет резонансную частоту. 372 Гц. Резистор имеет сопротивление 440 Ом, а конденсатор имеет емкость 112 мФ.Пиковое напряжение в цепи составляет 28 В. Каков пиковый ток в цепи при переменный ток в цепи имеет частоту 372 Гц? Ответьте с точностью до двух знаков после запятой.

    Ответ

    Резонансная частота контура заявлена ​​равной 372 Гц.

    Вопрос требует определения пикового тока.

    Пиковая приложенная разность потенциалов составляет 28 В. Для определения пиковый ток, необходимо определить полное сопротивление цепи.

    Полное сопротивление цепи определяется выражением 𝑍 = 𝑅 + (𝑋 − 𝑋) ,  где 𝑅 — сопротивление цепи, 𝑋 — индуктивное сопротивление цепи, и 𝑋 — емкостное реактивное сопротивление цепи.

    Значения 𝑋 и 𝑋 не указаны. Эти значения можно найти по формулам для емкостного и индуктивного сопротивления цепи, 𝑋 = 12𝜋𝑓𝐶, 𝑋 = 2𝜋𝑓𝐿,  и резонансная частота контура, 2𝜋𝑓 = 1𝐿𝐶.

    Однако, поскольку контур колеблется на своей резонансной частоте, это должно быть так, что 𝑋 − 𝑋 = 0 и, следовательно, что 𝑍 = 𝑅.

    Чтобы найти наибольший ток, нам нужно только разделить наибольшую разность потенциалов на нем на: = 28440 Ом

    С точностью до двух десятичных знаков 𝐼 равно 0,06 А.

    Пример 3: Определение емкости цепи, колеблющейся на своей резонансной частоте

    Цепь, содержащая последовательно соединенные конденсатор и катушку индуктивности, имеет резонансную частоту. 575 кГц. Катушка индуктивности в цепи имеет индуктивность 1,25 Гн. Какая емкость конденсатора? Ответ дайте в научном представлении с двумя десятичными знаками.

    Ответ

    Емкость конденсатора связана с резонансной частотой цепи, которая определяется выражением 2𝜋𝑓 = 1𝐿𝐶, где указано как 1,25 H и указано быть 575 кГц, что составляет 5,75 × 10 Гц.

    Формулу нужно переставить так, чтобы 𝐶 стал предметом, следующим образом: (2𝜋𝑓) = 1𝐿𝐶𝐶 = 1 (2𝜋𝑓) 𝐿.

    Подставляя известные значения, получаем 𝐶 = 1 (2𝜋 × 5,75 × 10) × 1,25.HzH

    В экспоненциальном представлении с точностью до двух десятичных знаков, 𝐶 составляет 6,13 × 10 F.

    Пример 4: Определение индуктивного сопротивления цепи, колеблющейся на своей резонансной частоте

    Цепь, в которой последовательно соединены конденсатор и катушка индуктивности, имеет резонансную частоту. 155 кГц. Конденсатор в цепи имеет емкость 215 мкФ. Какое индуктивное сопротивление цепи? Ответ дайте в научном представлении с двумя десятичными знаками.

    Ответ

    Индуктивное сопротивление цепи определяется выражением 𝑋 = 2𝜋𝑓𝐿,  где заявлено равным 155 кГц, что равно 1.55 × 10 Гц, и 𝐿 — индуктивность катушки индуктивности.

    Стоимость 𝐿 не указана. Однако его можно найти, используя резонансную частоту контура, который дается 2𝜋𝑓 = 1𝐿𝐶, где 𝐶 заявлено как 215 мкФ, что составляет 2,15 × 10 F.

    Формулу для резонансной частоты необходимо изменить так, чтобы 𝐿 стал субъектом, следующим образом: (2𝜋𝑓) = 1𝐿𝐶𝐿 = 1 (2𝜋𝑓) 𝐶.

    Подставляя известные значения, получаем 𝐿 = 1 (2𝜋 × 1,55 × 10) × 2,15 × 10𝐿≈4,9 × 10 HzFH

    Мы можем подставить это значение в 𝑋 = 2𝜋𝑓𝐿.

    В экспоненциальном представлении с точностью до двух десятичных знаков, составляет 4,78 × 10 Ом.

    Этот вопрос также можно решить, вспомнив, что на резонансной частоте цепи емкостные и индуктивные реактивные сопротивления равны, поэтому вместо индуктивного реактивного сопротивления можно было бы рассчитать емкостное реактивное сопротивление. Емкостное реактивное сопротивление можно определить просто по частоте и емкости цепи путем перестановки формулы 𝑋 = 12𝜋𝑓𝐶,  поскольку значения 𝑓 и 𝐶 оба указаны в вопросе.

    Давайте теперь подведем итог тому, что было изучено в этом объяснителе.

    Ключевые моменты

    • Резонансная частота последовательной цепи переменного тока, имеющей индуктивность и емкость, является частотой при которой приложенная переменная разность потенциалов генерирует наибольший ток.
    • На резонансной частоте цепи переменного тока полное сопротивление цепи равно сопротивлению цепи.
    • Резонансная частота цепи переменного тока определяется выражением 2𝜋𝑓 = 1𝐿𝐶, где 𝐿 — индуктивность цепи, а 𝐶 — емкость цепи.

    Индуктивность переменного тока и индуктивное реактивное сопротивление

    Введение

    В предыдущем руководстве, посвященном сопротивлению переменному току, мы видели, что поведение резистора одинаково в режиме постоянного или переменного тока при нормальных частотах. Однако другие основные электрические компоненты, такие как индуктор , , сильно зависят от частоты сигнала переменного тока.

    Представление концепции индуктивности дается в первом разделе, чтобы понять явления, которые определяют, как работает индуктор.Описание катушек индуктивности также дается позже в этом же разделе. Мы также объясняем, почему определенные типы индукторов лучше подходят для высокочастотных приложений.

    Во втором разделе подчеркивается зависимость от частоты индуктивности и наблюдаемого в них фазового сдвига.

    Наконец, последний раздел посвящен объединению резистора и катушки индуктивности для создания схем фильтрации.

    Презентация

    Индуктивность

    Индуктивность — это свойство электрического компонента, которое заключается в создании противодействия изменению тока, когда на его выводы подается переменное напряжение.Это замедление тока можно объяснить электромагнитным законом индукции , который подробно описан в руководстве по сопротивлению переменному току.

    Чтобы лучше понять это определение, давайте рассмотрим схему, которая состоит из источника переменного тока и никаких других компонентов, кроме проводящего провода длиной l , закорачивающего источник, как показано на Рис. 1 ниже:

    рис. 1: Источник переменного тока, приложенный к однополярной проводке.

    Токовая петля, образованная цепью, создает магнитное поле B , когда в нее протекает ток I , генерируемый источником переменного тока.Кружок с крестом обозначает, что при протекании тока I, как показано на рис. , рис. 1, , магнитное поле направлено вниз.

    Согласно закону Ленца, если ток имеет форму переменного тока, B также является переменным, что создает индуцированный ток I i , генерируемый электродвижущей силой E , чтобы смягчить изменение магнитное поле с индуцированным полем B i . Когда ток I i течет, как показано на Рис. 1 , индуцированное магнитное поле направлено вверх, что обозначено кружком с точкой.

    Выражение магнитного поля B дается следующим Уравнение 1 :

    уравнение 1: Выражение магнитного поля внутри однопроволочного

    , где μ 0 = 4π × 10 -7 Гн / м — это магнитная проницаемость вакуума . Следовательно, магнитный поток равен Φ = B × A , где A — поперечное сечение проволочной петли. Наконец, индуктивность L определяется по формуле Уравнение 2 ниже:

    уравнение 2: Определение индуктивности однопроводного кабеля

    Следовательно, индуктивность зависит только от геометрии, а не от источника (напряжения или тока), и выражается в Генри (H).Чтобы лучше понять, что представляет собой эта величина, стоит дать выражение электродвижущей силы E в Уравнение 3 :

    уравнение 3: Выражение электродвижущей силы

    Если мы выделим L в этом уравнении, мы поймем, что 1 Генри представляет индуктивность цепи, в которой электродвижущая сила в 1 вольт индуцируется при переменном токе со скоростью 1 ампер в секунду. втекает в него.

    Катушки индуктивности

    Чтобы значительно увеличить индуктивность провода, было бы очень сложно работать только с геометрическими параметрами A и l .На практике компоненты индуктивности называются индукторами , которые представляют собой провода с определенным числом витков вокруг оси и представлены в виде электрического компонента, например, как показано на Рис. 2 :

    Рис 2: Представление индуктора с воздушным сердечником

    Если мы рассмотрим количество N обмоток, Уравнение 2 изменится на Уравнение 4 :

    уравнение 4: Выражение индуктивности катушки индуктивности

    Намотанный провод с Н витками , следовательно, может видеть, что его индуктивность увеличивается в Н 2 раз по сравнению с тем же проводом, представленным на Рис.

    Есть еще одна модификация, которую можно сделать с индукторами для увеличения их индуктивности: добавить сердечник . Сердечник представляет собой центральную часть индуктора, окруженную проводом.

    Он может быть изготовлен из немагнитного материала, такого как пластик, керамика или даже воздух, который по своим магнитным свойствам аналогичен вакууму. Эти типы индукторов известны как индукторы с воздушным сердечником и представлены символически, например, на , рис. 2, , они имеют меньшую индуктивность, но лучшие частотные характеристики.

    Другой тип — это индукторы с ферромагнитным сердечником , их сердечник сделан из магнитного материала, который увеличивает общую магнитную проницаемость. Индукторы с ферромагнитным сердечником имеют гораздо более высокую индуктивность, чем индукторы с воздушным сердечником, потому что они могут лучше концентрировать магнитное поле. Однако они менее подходят для высокочастотных применений из-за важного присутствия Вихревых токов в сердечнике при увеличении частоты, что приводит к потерям из-за тепла.

    Катушки индуктивности с ферромагнитным сердечником могут быть представлены следующим образом:

    Рис 3: Изображение индуктора с ферромагнитным сердечником

    Частотная характеристика

    Реактивное сопротивление

    Довольно двусмысленно говорить об «индуктивности переменного тока», поскольку понятие индуктивности справедливо только в режиме переменного тока. Действительно, рассматривая уравнение 3 и сказанное в предыдущем разделе, можно понять, что электродвижущая сила, индуцированный ток и магнитное поле существуют только при наличии источника переменного тока.Фактически, в режиме постоянного тока индуктор просто рассматривается как простой провод, укорачивающий часть цепи.

    Импеданс катушки индуктивности можно определить как противодействие току, который генерирует индуктивный компонент. Импедансы, как объяснено в другом руководстве, представляют собой комплексные числа, у которых действительная часть представляет сопротивление , а мнимая часть представляет реактивное сопротивление .

    Для катушки индуктивности реактивное сопротивление X L представляет сопротивление компонента изменению тока.Импеданс катушки индуктивности является чисто мнимым числом, что означает, что сопротивление не учитывается, а его реактивное сопротивление выражается в следующей формуле:

    уравнение 5: Реактивное сопротивление катушки индуктивности

    Противодействие переменному току увеличивается линейно с частотой. При f = 0 Гц (режим постоянного тока) индуктор, как было сказано ранее, действует как короткое замыкание, а когда f → + ∞, он действует как разомкнутая цепь.

    Фазовый сдвиг

    Импеданс катушки индуктивности просто определяется как Z L = jX L .Фазовый сдвиг φ между током на катушке индуктивности и напряжением, генерируемым на его выводах (электродвижущая сила), является постоянным и определяется как Arg (Z L ) = arctan (y) при y → + ∞, что дает φ = + π / 2 рад или 90 ° .

    Это конкретное значение фазового сдвига, как подчеркивается в учебном пособии по фазоделителям, называется квадратурным опережением фазы и проиллюстрировано на Рис. 3 :

    Рис. 3: Иллюстрация квадратурного фазового сдвига в катушке индуктивности.Построено с помощью MatLab®

    .Если амплитуда (относительно пика) тока определяется как I max , амплитуда электродвижущей силы составляет В max = I max × X L .

    Явления индукции

    Когда частота источника переменного тока увеличивается, внутри катушек индуктивности происходят аналогичные явления, которые уже были подробно описаны в руководстве по сопротивлению переменному току.

    Первый — это скин-эффект , он вызван перераспределением тока около границы провода, составляющего катушку индуктивности.Эта высокая концентрация тока в небольшой области увеличивает сопротивление индуктора, что, как следствие, приводит к тепловым потерям.

    Эффект близости также присутствует в индукторах. Действительно, каждый виток провода наводит вихревые токи в пограничной области соседних проводов. Как и в случае скин-эффекта, это имеет тенденцию к уменьшению эффективного поперечного сечения для тока, что увеличивает сопротивление.

    Фильтры RL

    В этом руководстве мы видели, что индукторы очень чувствительны к частоте, а резисторы — нет.Возникает естественный вопрос: : что произойдет, если мы объединим эти компоненты в одну схему?

    Давайте рассмотрим схему RL, входное напряжение которой указано в V в , а выходное напряжение V на выходе :

    . Рис. 4: Последовательный контур RL

    Мы называем T V передаточной функцией схемы, которая определяется соотношением V out / V in . Чтобы найти выражение T V , мы можем просто посмотреть на схему как на делитель напряжения.Как следствие имеем:

    уравнение 6: Выражение передаточной функции цепи RL

    Эта функция интересна тем, что дает нам одновременно коэффициент усиления по напряжению и фазовый сдвиг цепи для любой частоты.

    Коэффициент усиления задается модулем T V , а фазовый сдвиг Φ — его аргументом, оба приведены в Уравнении 7 ниже. Мы рекомендуем читателю обратиться к руководству по комплексным числам, чтобы узнать, как определить модуль и аргумент комплексной дроби.

    уравнение 7: Коэффициент усиления и фазовый сдвиг цепи RL

    Возьмем R = 1 кОм и L = 100 мГн и построим график двух величин, представленных в уравнении Уравнение 7 , в логарифмическом масштабе для частоты. Результирующий график обычно известен как диаграмма Боде и показан на Рис. 5 :

    . Рис. 5: Диаграмма Боде цепи RL

    При низкой частоте (режим постоянного тока) импеданс индуктивности незначителен, поэтому падение напряжения полностью поглощается сопротивлением, а не индуктивностью.В режиме постоянного тока наблюдается квадратурный фазовый сдвиг.

    На высокой частоте преобладает импеданс индуктивности, и поэтому падение напряжения поглощается индуктивностью, что дает коэффициент усиления, стремящийся к 1. Сдвиг фазы стремится к нулю при увеличении частоты.

    В заключение можно сказать, что схема RL действует как фильтр верхних частот , поскольку низкие частоты не передаются.

    Заключение

    Как было показано в этом руководстве, катушки индуктивности при изменении частоты ведут себя иначе, чем резисторы.

    Прежде всего, было дано представление о концепции индуктивности. Индуктивность является важным фактором для понимания закона индукции, она описывает, насколько сильно компонент будет реагировать на изменение тока в цепи. Позже, представив катушки индуктивности, мы увидели, что индуктивность может быть увеличена с помощью обмоток и наличия магнитопровода.

    Во втором разделе мы более внимательно рассмотрели частотные характеристики катушек индуктивности.Их импеданс линейно увеличивается с частотой, в режиме постоянного тока они ведут себя как короткие замыкания и становятся разомкнутыми цепями, когда частота очень высока.

    Комбинирование катушек индуктивности с резисторами было исследовано в последнем разделе. Мы показали, как описать поведение RL-фильтра с помощью концепции передаточной функции и диаграммы Боде.

    .
    Разное

    Добавить комментарий

    Ваш адрес email не будет опубликован. Обязательные поля помечены *